packrat 13

83
Copyright © 2008. Physician Assistant Education Association 1 ACKNOWLEDGEMENT The time and effort provided by the following individuals who served as members of this committee are greatly appreciated: Kimberly Cavanagh, MPAS, PA-C, Project Director Mark Archambault, DHSc, MHS, RPA-C Jonathan Bowser, MS, PA-C Petar Breitinger, MPAS, PA-C Christine Bruce, MHSA, PA-C Denise Dillingham, MPAS, PA-C Kathleen Ehrhardt, MMS, PA-C Ralph Rice, MPAS, PA-C Eric Vangsnes, PhD, PA-C James VanRhee, MS, PA-C Donna Yeisley, MEd, PA-C Subcommittee Rex Hobbs, MPAS, PA-C Alan Gindoff, DHSc, PA-C Ryan Junsay, MS, PA-C Marc Maller, MD DEDICATION This examination would not have been possible without the years of commitment of the MR. TIB Development Committee. Numerous PA educators from across the nation provided their experience and insight as questions for MR. TIB. It has been this data bank that served as the building blocks for PACKRAT. PAEA is proud to be able to continue in the tradition of quality fostered by the forerunners of the self- assessment examination for physical assistants. It is our honor to dedicate PACKRAT to: Jesse C. Edwards, MS Claire S. Parker, PhD University of Nebraska, Physician Assistant Program

Upload: deb-haart

Post on 17-Feb-2016

12 views

Category:

Documents


2 download

TRANSCRIPT

Copyright © 2008. Physician Assistant Education Association 1

ACKNOWLEDGEMENT The time and effort provided by the following individuals who served as members of this committee are greatly appreciated:

Kimberly Cavanagh, MPAS, PA-C, Project Director Mark Archambault, DHSc, MHS, RPA-C

Jonathan Bowser, MS, PA-C Petar Breitinger, MPAS, PA-C Christine Bruce, MHSA, PA-C

Denise Dillingham, MPAS, PA-C Kathleen Ehrhardt, MMS, PA-C

Ralph Rice, MPAS, PA-C Eric Vangsnes, PhD, PA-C James VanRhee, MS, PA-C Donna Yeisley, MEd, PA-C

Subcommittee Rex Hobbs, MPAS, PA-C Alan Gindoff, DHSc, PA-C Ryan Junsay, MS, PA-C

Marc Maller, MD

DEDICATION This examination would not have been possible without the years of commitment of the MR. TIB Development Committee. Numerous PA educators from across the nation provided their experience and insight as questions for MR. TIB. It has been this data bank that served as the building blocks for PACKRAT. PAEA is proud to be able to continue in the tradition of quality fostered by the forerunners of the self-assessment examination for physical assistants. It is our honor to dedicate PACKRAT to:

Jesse C. Edwards, MS Claire S. Parker, PhD

University of Nebraska, Physician Assistant Program

Copyright © 2008. Physician Assistant Education Association 2

PHYSICIAN ASSISTANT EDUCATION ASSOCIATION

Physician Assistant Clinical Knowledge Rating and Assessment Tool (PACKRAT)

Form 13

Directions and Explanations

TABLE OF CONTENTS

I. Introduction 1

II. Explanation of the Score Report 2

Page 1: Scores – Your total Score and Group Comparisons 2 Page 2: Your STRENGTHS, Weaknesses, and Quality of Responses 2 Page 3: Your Individual and Correct Responses 2 Page 4: Your Responses by Task and Category 2 Page 5-6: Your Profile Comparison: Demographic Profile 2

III. Recommendations for Using the Feedback Package 3

IV. Study Resources 5

V. Answer Key 7

VI. Examination Explanations 8

VII. Comment Form 82

Copyright © 2008. Physician Assistant Education Association 3

Copyright © 2008. Physician Assistant Education Association. All rights reserved. No part of this publication may be reproduced or transmitted in any form or by any means, electronic or mechanical, including photocopy or recording, or any information and retrieval system, without permission in writing from the Physician Assistant Education Association.

PHYSICIAN ASSISTANT CLINICAL KNOWLEDGE RATING AND ASSESSMENT TOOL (PACKRAT)

I. Introduction The Physician Assistant Clinical Knowledge Rating and Assessment Tool (PACKRAT) was developed by a volunteer committee of experts and is based on the content outline of a nationally recognized competency examination. The following is a description of the content of PACKRAT:

PACKRAT EXAMINATION MATRIX CONTENT AREA NUMBER OF ITEMS 1. History & Physical 26 2. Diagnostic Studies 36 3. Diagnosis 45 4. Health Maintenance 22 5. Clinical Intervention 32 6. Clinical Therapeutics 48 7. Scientific Concepts 16 TOTALS 225 Additionally, questions also apply to the following clinical specialties: A. Cardiology I. Neurology B. Dermatology J. Obstetrics/Gynecology C. Endocrinology K. Orthopedics/Rheumatology D. ENT L. Pediatrics E. Ophthalmology M. Psychiatry/Behavioral Medicine F. Gastrointestinal/Nutritional N. Pulmonology G. Geriatrics O. Surgery H. Hematology P. Urology/Renal The task and specialty categories for each item are listed in the answer key on page 5; your feedback package contains a breakdown of responses by the task and clinical specialty category. Pay particular attention to the questions you answered incorrectly and determine the specialty for that question and use this information to identify weaknesses.

PACKRAT EXAMINATION MATRIX

CONTENT AREA NUMBER OF ITEMS 1. History & Physical 36 2. Diagnostic Studies 32 3. Diagnosis 41 4. Health Maintenance 22 5. Clinical Intervention 31 6. Clinical Therapeutics 41 7. Scientific Concepts 22 TOTALS 225 Additionally, questions also apply to the following clinical specialties: A. Cardiology J. Obstetrics/Gynecology B. Dermatology K. Orthopedics/Rheumatology C. Endocrinology M. Psychiatry/Behavioral Medicine F. Gastrointestinal/Nutritional N. Pulmonology

H. Hematology P. Urology/Renal I. Neurology Q. Infectious Diseases

Copyright © 2008. Physician Assistant Education Association 4

The PACKRAT provides a detailed feedback report of performance and it is available to anyone at any time. Explanations were developed for all the questions to provide a rationale for correct, as well as incorrect, answers. This information will help determine strengths and weaknesses with respect to the PACKRAT content outline. If you have weaknesses in specific areas, you may need to obtain additional clinical experience in those areas. This booklet is designed to explain and interpret the information contained in the accompanying computerized score report. You can use the report package to learn more about your abilities.

II. Explanation of the Score Report This section provides an interpretation of each page of the computerized score report you received. You should have your computer score report in front of you. Begin on page 1 of the report and read the following information. Page 1: Scores – Your Total Score and Group Comparisons Page 1 is an overview of the PACKRAT feedback report. Toward the bottom of the page is your examination score. This score shows the number of questions you answered correctly out of a possible 225. The average score for all first-year and second-year candidates who have taken the PACKRAT to date is also given. Page 2: Your Strengths, Weaknesses, and Quality of Responses Page 2 of the score report gives an overview of the content area in which your performance is categorized as “Strong”, “Satisfactory”, or “Needing Improvement”. These areas are based on the examination matrix on page 1. In each content area, your answers have been classified as correct, acceptable, unsatisfactory, or harmful. A definition of these classifications is also provided on this page. Pay particular attention to the areas under “Needing Improvement”, as these areas should be noted for further study. Also check the answer key for the specialty area of these items. If you selected a harmful answer in any content area, it will be automatically placed in the “Needing Improvement” category, regardless of the number of correct answers selected. Carefully review these questions and their explanations and specialty classifications in Section VI to help you understand why your answers were correct.

Page 3: Your Individual and Correct Responses Page 3 lists your answers to all questions. When your answer differs from the correct one, the proper response appears in parentheses. Use Section VI with this page to review the rationale for each option that is provided in the explanations, which are referenced to the study resources. The explanations may help you understand why one answer is more appropriate than another, or not the best answer, and why some of your answers may have been incorrect. If the option you chose was judged potentially harmful to the patient or others, an asterisk (*) appears before your answer. Options classified as potentially harmful may identify serious weaknesses. Go over these questions carefully and read the explanations for the correct answers. You may be able to identify areas where you need further study. Page 4: Your Responses by Task and Specialty Category Page 4 lists your responses by both specialty and task category. You will be able to identify the areas of the content outline where you may have difficulty. The numbers reflect how many items you answered correctly out of the total possible correct within each task and specialty area. Categories 1 through 7 identify the task areas and A-P the clinical specialty areas. If you missed a significant number of items in an area, check the key and go over the explanations for the items in these areas. Page 5-6: Your Profile Comparison: Demographic Profile Page 5 is the beginning of the Demographic Profile Comparison for programs. This profile shows the reported demographic information and compares your students’ information to the entire group of individuals who have taken the PACKRAT to date. The demographic data

Copyright © 2008. Physician Assistant Education Association 5

are based on the information your students provided when answering the demographics questions. The summary demographic information shown reflects all the data compiled for either first or second-year students who taken the PACKRAT. RESPONDENT DEMOGRAPHIC INFORMATION C. Number of months of clinical rotations completed at the time of this exam? Your response: All Second-year Respondents: ( ) 1. None ( 0% ) ( ) 2. Less than 3 months ( 5% ) ( ) 3. 3 to 6 months ( 15%) ( ) 4. 7 to 9 months ( 20%) ( ) 5. 10 to 12 months (45%) ( ) 6. Greater than 12 months (15%) Programs can utilize this information in order to compare the characteristics of their first year students and second year students to the national demographics of first year and second year students who have taken the PACKRAT test. III. Recommendations for using The Feedback Package As a current physician assistant student, PACKRAT can be a useful self-evaluation tool. Through careful review of question explanations, noting specific tasks and content areas, you will be able to assess your current strengths and weaknesses. You will be able to identify particular areas in which to concentrate more effort as you continue your studies. By concentrating your effort on the areas in which you did not do well, you may improve your performance, and you may have a better chance of passing the proctored examination. However, PAEA cannot guarantee that this will occur, since the conditions under which you attempted the PACKRAT may have been different from those in a standardized administration of a proctored examination. Use the explanations in Section VI to analyze why you chose various options. Again, pay particular attention to the options that were judged potentially harmful or unsatisfactory. Look at the question and the four options

again to see why the answer you chose was incorrect. If there appears to be a deficit in your exposure to a particular clinical specialty, perhaps further study would make you more familiar with these situations. Once you have completely reviewed your score report and this booklet, PAEA hopes you will use this information to improve your overall performance, either on the job or on future certification examinations. Should you wish to provide suggestions about PACKRAT to PAEA, you will find a comment form on the last page of this booklet. IV. Study Resources A variety of textbooks are currently available to assist candidates in preparing for the certification examination. For additional information, you may contact a faculty member at an educational program or an experienced colleague if you need help determining which references to review in a specific content area. A short list of general textbooks is below. All examination questions are related to material found in these resources. Please note that the books on this list are not available from PAEA. This is not intended as an all-inclusive list, and the materials listed below are suggested study materials only.

1. Andreoli TE, et al (eds). Cecil’s Essentials of Medicine. 7th ed. Philadelphia, PA: WB Saunders, an Elsevier company, 2007

2. Ballweg R et al. Physician Assistant: A Guide to Clinical Practice. 3rd ed., WB Saunders, an Elsevier company, 2003. ISBN#0721600174

3. Bickley, LS. Bates’ Guide to Physical Examination and History Taking. 9th ed. Philadelphia, PA: JB Lippincott Co., 2007

4. Beckmann CR, et al. Obstetrics & Gynecology. 5th ed. Philadelphia, PA: Lippincott Williams & Wilkins, 2006.

5. Behrman RE, et al. Nelson’s Textbook of Pediatrics. 18th ed. Philadelphia, PA: WB Saunders, an Elsevier company, 2007

6. Berkowitz, C. Pediatrics: A Primary Care Approach, 2nd ed. Philadelphia, PA: WB Saunders, an Elsevier company 2000.

Copyright © 2008. Physician Assistant Education Association 6

7. Fauci AS, et al ( eds). Harrison’s Principles of Internal Medicine. 16th ed. New York, NY: McGraw-Hill, Inc., 2005.

8. DeCherney AH & Pernoll ML (eds.) Current Obstetric & Gynecological Diagnosis & Treatment, 10th ed., Norwalk, CT: Appleton & Lange, 2006

9. Ellsworth AJ, et al. (eds). Mosby’s Medical Drug Reference. Philadelphia, PA: Mosby, 2007.

10. Wolf K, Johnson RA, and Surmond D. Fitzpatrick’s Color Atlas and Synopsis of Clinical Dermatology. 5th ed. New York, NY: McGraw-Hill, Inc., 2005.

11. Goldman J and Bennet JC. Cecil Textbook of Medicine. 23rd ed. Philadelphia, PA: WB Saunders. 2008.

12. Goroll, AH, Mulley, AG & May, LA. Primary Care Medicine: Office Evaluation and Management of the Adult Patient. 5th ed., Lippincott Williams &Wilkins, 2005

13. Hay WW, et al. Current Pediatric Diagnosis and Treatment. 18th ed. Norwalk, CT: McGraw Hill, 2005.

14. Kaplan HI and Sadock BJ (eds). Synopsis of Psychiatry. 10th ed., Philadelphia, PA: Williams & Wilkins, 2007.

15. Katzung BG. Basic and Clinical Pharmacology. 10th ed. Stamford, CT: Appleton & Lange, 2006.

16. Mandel GL, Bennett JE, and Dolin R. Principles and Practice of Infectious Disease. 6th ed., Philadelphia, PA: Churchill Livingston, an Elsevier company, 2005. ISBN#0443066434.

17. Marx, J., Hockberger, RS, and Walls, RM. Rosen’s Emergency Medicine: Concepts and Clinical Practice. 6th ed., Philadelphia, PA: Mosby, 2005.

18. McPhee SJ, et al. Pathophysiology of Disease. 5th ed., McGraw Hill, 2005.

19. Mercier LR, et al. Practical Orthopedics. 6th ed. Philadelphia, PA: Mosby, an Elsevier company, 2008. ISBN#9780323036184

20. Mettler FA, et al. Primary Care Radiology. Philadelphia, PA: WB

Saunders, Co., 2000. (In PAEA library 2/07)

21. Howland RD and Mycek MJ. Lippincott’s Illustrated Reviews: Pharmacology. 3rd ed. Baltimore, MD: Williams & Wilkins, 2005.

22. Noble J, et al. Textbook of Primary Care Medicine. 3rd ed. Philadephia PA: Mosby, 2001. (In PAEA library 2/07)

23. Riordan-Eva, P. and Whitcher, JP. Vaughn & Asbury’s General Ophthalmology. 17th ed., McGraw Hill, 2007.

24. Sacher RA and McPherson RA. Widmann's Clinical Interpretation of Laboratory Tests. 11th ed. FA Davis Co., 2000. (In PAEA library 2/07)

25. Schwartz SI, et al. Principles of Surgery. 8th ed. New York, NY: McGraw-Hill, Inc., 2004.

26. Skinner HB (ed) Current Diagnosis & Treatment in Orthopedics. 4th ed., Norwalk,CT:Appleton & Lange, 2008.

27. Steinberg GG. Orthopedics in Primary Care. 3rd ed. Philadelphia, PA: Lippincott Williams & Wilkins, 3rd ed, 1999.

28. Tierney LM, et al. Current Medical Diagnosis and Treatment. 47th ed. Stamford, CT: Appleton & Lange, 2008.

29. Tintinalli JE, Kelen GD, and Stapezynski JS. Emergency Medicine: A Comprehensive Study Guide. 6th ed. New York, NY: McGraw-Hill, Inc., 2004.

30. Townsend CM. Sabiston’s Textbook of Surgery. The Biological Basis of Modern Surgical Practice. 18 ed. Philadelphia, PA: WB Saunders, an Elsevier company, 2008.

31. Doherty GM. Current Surgical Diagnosis and Treatment. 12th ed., McGraw Hill, 2005.

32. Wilson WR. Current Diagnosis and Treatment in Infectious Disease. Norwalk, CT: Appleton & Lange, 2001.

Copyright © 2008. Physician Assistant Education Association 7

THIS PAGE INTENTIONALLY LEFT BLANK FOR ANSWER

KEY

8

PHYSICIAN ASSISTANT EDUCATION ASSOCIATION

Physician Assistant Clinical Knowledge Rating and Assessment Tool

(PACKRAT) Form 13

EXPLANATIONS

1. Scientific Concepts/Cardiology Which of the following factors in patients with chronic venous insufficiency predisposes them to development of skin ulcers?

A. Increased intravascular oncotic pressure B. Leakage of fibrinogen and growth factors into the interstitial space C. Decreased capillary leakage D. Inherited deficiency of protein C Explanations (u) A. Decreased intravascular oncotic pressure can cause swelling (c) B. Leakage of fibrinogen and growth factors into the interstitial space, leukocyte aggregation and activation, and obliteration of the cutaneous lymphatic network can predispose a patient to skin ulcers (u) C. Increased capillary leakage causes venous insufficiency. (u) D. Inherited deficiency of protein C predisposes patients to thrombosis. Ref: (28) 2. History & Physical/Obstetrics/Gynecology

A 26 year-old monogamous female presents with cyclic pelvic pain that has been increasing over the last 6 months. She complains of significant dysmenorrhea and dyspareunia. She uses condoms for birth control. On physical examination her uterus is retroverted and non-mobile, and she has a palpable adnexal mass on the left side. Her serum pregnancy test is negative. Which of the following is the most likely diagnosis?

A. Ovarian cancer B. Endometriosis C. Functional ovarian cyst D. Pelvic inflammatory disease . Explanations (u) A. It is important to consider ovarian cancer in a patient with a pelvic mass however, ovarian cancer usually occurs in older women over age 55 and patients are often asymptomatic until the disease is more advanced (c) B. With endometriosis, the uterus is often fixed and retroflexed in the pelvis. The palpable mass is an endometrioma or "chocolate cyst". The patient with endometriosis also often has dysmenorrhea, dyspareunia, and dyschezia. (u) C. Functional ovarian cysts occur from ovulation and usually are not symptomatic. (u) D. With PID the patient will have abdominal tenderness, adnexal tenderness, cervical motion tenderness and an elevated temperature. Ref: (4)

9

3. Health Maintenance/ENT/Ophthalmology At what age does the first tooth usually erupt in an infant?

A. 2-4 months B. 6-8 months C. 10-12 months D. 14-16 months Explanations (u) A. See B for explanation. (c) B. The first tooth in an infant to erupt is the central incisor at the average age of 6-8 months. (u) C. See B for explanation. (u) D. See B for explanation. Ref: (3) 4. Clinical Therapeutics/Urology/Renal A 7 year-old boy wets the bed on most nights. Which of the following is the preferred pharmacological agent to decrease the incidence of bed wetting episodes? A. Imipramine (Tofranil) B. Phenytoin (Dilantin) C. Pramipexole (Mirapex) D. Hyoscyamine (Urised) Explanations (c) A. Imipramine is an anti-cholinergic and when given before bedtime has been shown to decrease the incidence of bed wetting. (u) B. Phenytoin is an anticonvulsant and is not used in enuresis. (u) C. Pramipexole is a dopamine agonist used in the treatment of restless leg syndrome. (u) D. Hyoscyamine is an anti-spasmodic used to treat overactive bladder. Ref: (5) 5. Diagnosis/Cardiology A newborn is being evaluated for perioral cyanosis while feeding associated with sweating. Vital signs are rectal temperature, 37.8 degrees C (100 degrees F), blood pressure 80/45 mmHg, pulse 180/min, and respirations 40/min. A grade 3/6 harsh systolic ejection murmur with a single loud S2 is heard at the left upper sternal border. Electrocardiogram (ECG) shows right ventricular hypertrophy with right axis deviation. Chest x-ray shows a boot-shaped heart and decreased pulmonary vascular markings. Which of the following is the most likely diagnosis? A. Atrial septal defect B. Total anomalous pulmonary venous return C. Coarctation of the aorta D. Tetralogy of Fallot Explanations (u) A. Although the murmur may be consistent with an ASD with pulmonary hypertension the chest x-ray would not show decreased pulmonary vascular markings. With a large left to right shunt large pulmonary arteries and increased vascularity would be seen. (u) B. The murmur for TAPVR is a soft systolic murmur at the left upper sternal border with a split S2 in addition to a short mid-diastolic murmur at the low left sternal border. (u) C. Cyanosis is usually not the presenting sign for coarctation of the aorta. Infants may present with heart failure, ECG will show evidence of LVH. (c) D. This is a common presentation for tetralogy of fallot. Ref: (28)

10

6. Diagnostic Studies/Endocrinology Which of the following is considered to be the modality of choice for the identification of a pituitary macroadenoma that is suspected on the basis of a visual field deficit? A. Skull x-ray B. PET scan C. CT of the brain D. MRI of the brain Explanations (u) A. Skull x-ray is not the modality of choice for the identification of pituitary macroadenoma. (u) B. PET scan is not the modality of choice for the identification of pituitary macroadenoma. (u) C. CT of the brain is not the optimal imaging technique for evaluation of the pituitary stalk. (c) D. MRI of the brain provides the best visualization of pituitary tumors. Ref: (28) 7. Clinical Intervention/Infectious Diseases A 2 year-old male presents with a four day history of fever and general malaise. On examination the vitals reveal an oral temperature of 102 degrees F. The child appears to have rubor on the trunk which started one day prior to this visit. Physical examination reveals a maculopapular rash with defervescence. Which of the following is the most appropriate management at this time? A. Ibuprofen (Motrin) B. Aspirin C. Amoxicillin D. Valacyclovir (Valtrex)

Explanations (c) A. Motrin is indicated for management of the fever in Roseola infantum caused by the herpesvirus. (h) B. Aspirin is contraindicated in children for management of fever. (u) C. Amoxicillin is not indicated for viral infections. (u) D. Valacyclovir is indicated for Varicella-Zoster virus infection for decreasing the incidence of varicella pneumonia only. In healthy children valacyclovir is not indicated due to its marginal therapeutic benefit. Ref: (13) 8. Clinical Therapeutics/Orthopedics/Rheumatology A 42 year-old female experiences pain on the plantar surface of her left foot in the area of the third metatarsal head. The pain is associated with wearing tight shoes and is relieved by removing shoes. Examination reveals a palpable mass and reproduction of pain with deep palpation of the third intermetatarsal space. The patient has tried wearing wider shoes with metatarsal cushions and taking NSAIDS but her symptoms persist. What is the best therapeutic option at this point? A. Casting of the involved foot B. Physical therapy C. Steroid injection D. Surgical excision Explanations (u) A. Casting the foot in a patient with Morton's neuroma is not effective therapy. (u) B. Physical therapy has not been shown to be of benefit in treating Morton's neuroma. (c) C. Steroid injection is the treatment of choice for Morton's neuroma when conservative measures fail. (u) D. Surgical excision is recommended for treatment of Morton's neuroma only if conservative measures and steroid injection have failed. Ref: (26)

11

9. History & Physical/Pulmonology Which of the following is the most likely to develop into a persistent cough in the adult patient? A. Pertussis B. Allergic rhinitis C. Pharyngitis D. Heart failure Explanations (c) A. Pertussis is suspected in patients with persistent cough that lasts longer than 2-3 weeks. Allergic rhinitis, pharyngitis and heart failure are all potential causes of acute cough. (u) B. See A for explanation. (u) C. See A for explanation. (u) D. See A for explanation. Ref: (28) 10. Diagnosis/Gastrointestinal/Nutritional A 30 year-old patient presents with weight loss, diarrhea, and steatorrhea. Labs reveal that the antiendomysial antibody (AEA) is positive. What is the most likely diagnosis? A. Celiac sprue B. Ulcerative colitis C. Whipple's disease D. Zollinger-Ellison syndrome Explanations (c) A. Celiac sprue is not only characterized by these classic symptoms. The antiendomysial antibody has a 90-95% sensitivity and 90-95% specificity for celiac sprue. (u) B. Ulcerative colitis could be responsible for the symptoms mentioned however antiendomysial antibody (AEA) would not be positive. (u) C. Whipple's disease is an infectious disorder known to cause diarrhea and weight loss. It also results in arthralgia and symptoms involving both the central nervous and cardiac systems that are not easily missed due to their severity. Diagnosis involves biopsies of the involved tissues looking for PAS-positive macrophages. (u) D. Zollinger-Ellison syndrome (ZE) is a hypersecretory disorder. Diarrhea can occur but will not generally be the predominant symptom. Ref: (7) 11. Clinical Intervention/Dermatology A patient sustained a 6 cm laceration on his anterior tibia that was primarily closed in the emergency department. What is the most appropriate time frame for removal of these sutures? A. 1-2 days B. 3-5 days C. 6-8 days D. 7-14 days Explanations (u) A. See D for explanation. (u) B. See D for explanation. (u) C. See D for explanation. (c) D. Suture removal is based upon the area of the body that was sutured. Facial sutures are placed for 3-4 days, scalp sutures for 5-7 days, trunk sutures are placed for 6-8 days, and sutures on the extremity are placed for 7-14 days. Sutures on the extremities can stay for longer periods of time if the area is under maximal tension. Ref: (2)

12

12. Clinical Therapeutics/Cardiology A hospitalized patient is found with confirmed pulseless ventricular tachycardia. IV access is obtained following the second shock given. Which of the following medications is to be administered immediately? A. Amiodarone B. Magnesium C. Atropine D. Epinephrine Explanations (u) A. Antiarrhythmics are given after the third shock and epinephrine has been administered. (u) B. Magnesium is useful for torsades de pointes. (u) C. Atropine may be used for asystole or a slow pulseless electrical activity (PEA) rate. (c) D. Epinephrine should be given as soon as IV access is obtained before or after the second shock. Ref: (11) 13. Diagnostic Studies/Psychiatry/Behavioral Medicine A patient with advanced AIDS complicated by toxoplasmosis presents with altered mental status, recent onset of seizures, and focal neurologic deficits. Which of the following diagnostic studies is most helpful? A. Toxoplasma gondii antibody titers B. CT scan of the brain C. Lumbar puncture D. MRI of brain Explanations (u) A. Antibody titers cannot be depended upon since most patients have IgG titers that reflect past infection, significant rises are infrequent, and IgM antibody is rare. (u) B. A CT scan may be helpful, but an MRI is the more sensitive test. (h) C. Lumbar puncture is contraindicated secondary to possible mass effect. (c) D. An MRI showing multiple isodense or hypodense ring-enhancing mass lesions is the most useful test for such a patient. Ref: (7) 14. Health Maintenance/Pulmonology A pediatric patient presents with a history of multiple recurrent respiratory infections associated with failure to thrive. A sweat chloride test is elevated. Which of the following is a common cause of death in patients with this condition? A. Diabetic ketoacidosis B. Pulmonary infection C. Intestinal obstruction D. Acute respiratory failure Explanations (u) A. While patients with cystic fibrosis most likely will eventually develop insulin-dependent diabetes mellitus, diabetic ketoacidosis is not a common cause of death. (c) B. This patient has cystic fibrosis. The most common causes of death include pulmonary complications, such as infections, and terminal chronic respiratory failure associated with cor pulmonale. (u) C. While intestinal obstruction may occur in patients with cystic fibrosis, it is not a common cause of death. (u) D. See B for explanation. Ref: (28) 15. History & Physical/Neurology In addition to tremor, which of the following are cardinal symptoms of Parkinson disease? A. Cognitive decline and rigidity B. Personality change and bradycardia C. Eye movement abnormalities and hyperkinesias D. Rigidity and bradykinesia

13

Explanations (u) A. See D for explanation. (u) B. See D for explanation. (u) C. See D for explanation. (c) D. Tremor, rigidity, bradykinesia and postural instability are the cardinal features of Parkinsonism and may be present in any combination. Ref: (28) 16. Scientific Concepts/Hematology Normal hemoglobin A is made of what combination of heme and globin chains? A. 2-alphas and 2-betas B. 4-gammas C. 2-alphas and 2-gammas D. 4-betas Explanations (c) A. Hemoglobin A1 is composed of two alpha and two beta chains. (u) B. Barts Hemoglobin-Hydrops Fetalis has 4-gamma chains and is incompatible with life. (u) C. Fetal hemoglobin consists of 2-alpha and 2-gamma chains. (u) D. Hemoglobin H consists of 4-beta chains and is nonfunctional. Ref: (28) 17. Clinical Intervention/Gastrointestinal/Nutritional Which of the following is the most important intervention in acute pancreatitis? A. IV fluid administration B. Antibiotic administration C. Calcium replacement D. Antiemetics Explanations (c) A. The mainstay of management in acute pancreatitis is fluid resuscitation. Isotonic solutions are best to maintain renal perfusion and urine output > 100 ml/hour. (u) B. 90% of patients will recover with supportive measures only. Parenternal antibiotics are not indicated in acute pancreatitis. (u) C. Hypocalcemia is a marker of prognosis, but correction of hypocalcemia does not supercede the importance of fluid resuscitation from the overwhelming inflammatory response. (u) D. Other supportive measures in acute pancreatitis include pain management and antiemetics, but the mainstay is fluid resuscitation. Ref: (29) 18. Diagnosis/Cardiology An electrocardiogram (ECG) shows a sinus rhythm with varying T wave heights, axis changes every other beat and a wandering baseline. Which of the following is most likely the diagnosis? A. Artifact B. Digoxin toxicity C. Pericardial effusion D. Poor lead placement Explanations (u) A. Artifact could show a wandering baseline, but not the distinct axis changes. (u) B. Digoxin toxicity can cause bidirectional tachycardia, but not electrical alternans. (c) C. This ECG pattern best represents pericardial effusion due to a swinging heart in fluid and is known as electrical alternans. (u) D. Poor lead placement would show different patterns per the leads. Ref: (28)

14

19. Clinical Therapeutics/Endocrinology Which of the following is the first-line treatment for a patient with mild syndrome of inappropriate secretion of ADH (SIADH)? A. Sodium supplementation to correct the hyponatremia B. Restriction of free water C. High volume hypertonic saline infusion D. Pituitary ablation via transsphenoidal approach Explanations (u) A. Patients with SIADH will have euvolemic hyponatremia but this is a water problem and not a sodium problem. Supplementation of additional sodium only helps with water retention in the body and can produce edema but it is not valuable in the correction of this hyponatremia. (c) B. Restriction of free water intake is the first-line therapy for patients with euvolemic hyponatremia that is caused by SIADH. Water intake should be restricted to 0.5 to 1 liter per day. A gradual increase in serum sodium will occur over days with this treatment. (h) C. High volume hypertonic saline infusion can result from too rapid correction of hyponatremia by causing central pontine myelinolysis. (h) D. Patients with SIADH are treated medically in order to increase the serum sodium rather than ablation of the pituitary gland which is not indicated for this condition. Ref: (28) 20. Diagnostic Studies/Pulmonology A 62 year-old homeless patient presents complaining of fever, weight loss, anorexia, night sweats and a chronic cough that recently became productive of purulent sputum that is blood streaked. On physical examination, the patient appears chronically ill and malnourished. Which of the following chest x-ray findings supports your suspected diagnosis? A. Hyperinflation and flat diaphragms B. Interstitial fibrosis and pleural thickening C. Cavitary lesions involving the upper lobes D. "Eggshell" calcification of hilar lymph nodes Explanations (u) A. Chest x-ray findings of hyperinflation and flat diaphragms suggest long-standing chronic obstructive lung disease. (u) B. Interstitial fibrosis and pleural thickening on a chest x-ray are found in cases of interstitial lung disease. (c) C. This patient most likely has tuberculosis. A chest x-ray finding of cavitary lesions involving the upper lobes would support this suspected diagnosis. (u) D. Chest x-ray findings of "eggshell" calcification of hilar lymph nodes strongly supports a diagnosis of silicosis. Ref: (1) 21. History & Physical/Orthopedics/Rheumatology Abduction of the shoulder against resistance helps localize pain in which of the following muscles of the shoulder girdle? A. Supraspinatus B. Infraspinatus C. Teres minor D. Subscapularis Explanations (c) A. Abduction against resistance tests the supraspinatus. (u) B. Lateral rotation against resistance tests the infraspinatus and teres minor. (u) C. See B for explanation. (u) D. Medial rotation against resistance tests the subscapularis. Ref: (4)

15

22. Health Maintenance/Obstetrics/Gynecology What is the recommended method for screening pregnant women for gestational diabetes? A. Fasting blood sugar and 2 hour post prandial B. 50 gram glucose load followed by a blood sugar in 1 hour C. 75 gram glucose load followed by a blood sugar in 2 hours D. 100 gram glucose load followed by a blood sugar at 1 hour, 2 hours, and 3 hours Explanations (u) A. Fasting blood sugar and 2 hour postprandial blood test is used to follow patient with gestational diabetes. (c) B. One hour Glucola is the screening test for gestational diabetes. It is a 50 gram glucose load, with a serum glucose obtained 1 hour after the dose. Normal value is less than 140 mg/dL. (u) C. A 75 gram glucose load is used in non-pregnant patients. (u) D. This describes a three-hour GTT, which is ordered if the 1 hour Glucola is elevated above 140 gm/dL. Ref: (4) 23. Diagnosis/ENT/Ophthalmology A 45 year-old smoker presents with a sore mouth and increasing difficulty eating for two weeks. Physical examination reveals a 1 cm white lesion on the buccal mucosa that cannot be rubbed off. Which of the following is the most likely diagnosis? A. Oral cancer B. Oral candidiasis C. Aphthous ulcer D. Necrotizing ulcerative gingivitis Explanations (c) A. The presence of leukoplakia in a smoker over the age of 40 should be biopsied to rule out the presence of oral cancer. (u) B. Oral candidiasis presents with white patches. Unlike leukoplakia, the patches easily rub off. (u) C. While aphthous ulcers are commonly found on the buccal mucosa, they are usually 1 to 2 mm round ulcerative lesions. (u) D. Necrotizing ulcerative gingivitis is common in young adults under stress. Clinically, it presents with painful acute gingival inflammation and necrosis. Ref: (28) 24. Clinical Therapeutics/Cardiology A 25 year-old female presents to the emergency department after an episode of substernal chest pain with radiation to the middle of her back that came on suddenly and lasted for about four minutes this morning while in bed. She denies previous episodes. Examination is unremarkable, but she appears jittery. Toxicology screen is positive for cocaine. Which of the following medications is contraindicated in this patient? A. Lorazepam (Ativan) B. Diltiazem (Cardizem) C. Nitroglycerin (Nitrostat) D. Propanolol (Inderal) Explanations (u) A. Lorazepam is not contraindicated and can help with agitation, psychosis or seizures. (u) B. Diltiazem is not contraindicated but does not have a definitive role in treating cocaine toxicity. (u) C. Nitroglycerin is not contraindicated but does not have a definitive role in treating cocaine toxicity. (c) D. Pure Beta blockers, such as propranol, can cause a paradoxical hypertension because of unopposed alpha-adrenergic effects. Ref: (28)

16

25. Scientific Concepts/Infectious Diseases Which immunoglobulin is the first to respond during the primary immune response for a gram positive bacterial infection? A. IgM B. IgG C. IgA D. IgE Explanations (c) A. IgM is the first immunoglobin to respond during the acute exposure. This immunoglobulin promotes opsonization and phagocytosis. IgG is the immunoglobulin that responds during the secondary exposure. IgE is the immunoglobulin that responds during an allergic response as well as during a parasitic infection. IgA is an antibody found in colostrums and GI secretions. (u) B. See A for explanation. (u) C. See A for explanation. (u) D. See A for explanation. Ref: (32) 26. History & Physical/Psychiatry/Behavioral Medicine A patient with obsessive compulsive disorder would most likely have which of the following findings? A. Raw, red hands B. Priapism C. Memory impairment D. Abdominal pain Explanations (c) A. Common manifestations of obsessive compulsive disorder include phobias of germ and contaminants which results in frequent handwashing leading to chafed and reddened hands. The other answers are inconsistent with obsessive compulsive disorder. (u) B. See A for explanation. (u) C. See A for explanation. (u) D. See A for explanation. Ref: (14) 27. Health Maintenance/Gastrointestinal/Nutritional As a rule, solid foods such as cereal and fruits are best introduced into an infant's diet at approximately A. 1 to 3 weeks. B. 4 to 8 weeks. C. 4 to 6 months. D. 10 to 14 months. Explanations (u) A. See C for explanation. (u) B. See C for explanation. (c) C. The inclusion of solid foods in the diet is best done at approximately 4-6 months. Before 4 to 6 months of age does not contribute significantly to the infant's health and it increases their risk of allergies and atopy. (u) D. After 6 months, meat, eggs, and starchy foods can be introduced. Ref: (13)

17

28. Clinical Intervention/Cardiology A 56 year-old female four days post myocardial infarction presents with a new murmur. On examination the murmur is a grade 3/6 pansystolic murmur radiating to the axilla. She is dyspenic at rest and has rales throughout all her lung fields. Blood pressure is 108/68 mmHg, pulse 70 bpm. Which of the following would be the definitive clinical intervention? A. Intra-aortic balloon counterpulsation B. Mitral valve replacement C. Coronary artery bypass surgery D. Immediate fluid bolus Explanations (u) A. Although part of the primary treatment to reduce mitral regurgitation, it is not definitive. (c) B. MVR is the definitive intervention to correct MR caused by papillary muscle rupture. (u) C. CABG may be necessary if significant blockage is found, but it will not correct the mitral regurgitation. (u) D. A fluid bolus is indicated if the patient is hypotensive. Ref: (28) 29. Diagnosis/Neurology During an influenza epidemic, a 6 year-old male is seen with fever and a severe sore throat. The parents report that his symptoms have not improved despite administration of aspirin. The next day, the parent calls to report that the child has persistent vomiting and increased lethargy. On examination, he is found to be delirious and disoriented with hyperactive reflexes. The liver edge is 3 cm below the right costal margin in the midclavicular line. Which of the following is the most likely diagnosis? A. Reye's syndrome B. Measles encephalitis C. Guillain-Barre syndrome D. Acute bacterial meningitis Explanations (c) A. The suspected influenza associated with development of vomiting, progressive mental status changes, hyperreflexia, and hepatomegaly are consistent with a diagnosis of Reye's syndrome. (u) B. Measles encephalitis is a complication of rubeola, which is not suggested by this patient's presentation. (u) C. Fever is uncommon with Guillain-Barre, which is also characterized by ascending symmetrical weakness of the lower extremities. (u) D. While acute bacterial meningitis might present with acutely evolving symptoms, hepatomegaly is not usually seen. Ref: (13) 30. Clinical Therapeutics/Obstetrics/Gynecology What is the treatment of magnesium sulfate toxicity? A. Nifedipine B. Terbutaline C. Potassium carbonate D. Calcium gluconate Explanations (u) A. Nifedipine, a calcium-channel blocker is used to treat both preterm labor and hypertension in pregnancy. It works by inhibiting calcium transport through slow-type channels, causing reduction in systemic and pulmonary vascular resistance and tocolysis. (u) B. Terbutaline is a beta-blocker that is used to treat pre-term labor. (u) C. Potassium carbonate is a treatment for metabolic acidosis, not magnesium sulfate toxicity. (c) D. 10% calcium gluconate is used to treat magnesium sulfate toxicity.

Ref: (4)

18

31. Diagnostic Studies/Orthopedics/Rheumatology A 26 year-old male who is an avid swimmer has been experiencing right shoulder pain for the past month. On examination, pain is elicited with palpation below the anterior acromion. Anterior shoulder pain is also reported when the patient flexes and extends his arm. Which of the following diagnostic tests is most appropriate at this time? A. Shoulder x-ray B. Shoulder arthroscopy C. Shoulder MRI D. Subacromial lidocaine injection Explanations (u) A. This patient is experiencing mild impingement syndrome and, although plain films may demonstrate bone spurs on the undersurface of the acromion, his age and relatively recent onset of symptoms suggest that this study would likely be normal. (u) B. Arthroscopy is not indicated unless less invasive modalities fail to aid in the diagnosis and symptoms of impingement persist despite therapy. (u) C. While MRI would likely support the diagnosis, it is an expensive modality and not necessary to diagnose mild impingement. (c) D. Subacromial injection of lidocaine leading to a transient but dramatic improvement in pain with shoulder extension makes the diagnosis of impingement highly likely. Ref: (27) 32. Scientific Concepts/Pulmonology Which of the following is the major pathogenetic mechanism that causes asthma? A. Airway inflammation B. Increased pulmonary secretions C. Presence of Ghon complexes D. Irreversible fibrosis Explanations (c) A. Airway inflammation is the major pathogenetic mechanism that leads to the development of asthma. (u) B. Increased pulmonary secretions are the mechanism in chronic bronchitis. (u) C. The presence of Ghon complexes is noted in pulmonary tuberculosis. (u) D. Irreversible fibrosis of the lung parenchyma is associated with interstitial lung diseases. Ref: (1) 33. History & Physical/ENT/Ophthalmology When performing a Weber test on a patient with impacted cerumen in the right canal, the sound should be A. referred to the right ear. B. referred to the left ear. C. equal in both ears. D. louder with air conduction. Explanations (c) A. In unilateral conductive hearing loss, the sound is referred to the impaired ear. (u) B. See A for explanation. (u) C. See A for explanation. (u) D. Bone conduction as noted with the Rinne test is louder than air with conductive hearing loss. Ref: (3)

19

34. Health Maintenance/Urology/Renal Which of the following is used to monitor possible recurrence of prostate cancer? A. Prostate specific antigen B. Acid phosphatase C. Transrectal ultrasound D. Bone scan Explanations (c) A. Increasing levels of prostate specific antigen are consistent with progression of disease. (u) B. Acid phosphatase levels may increase in prostate cancer, but are not as sensitive as prostate specific antigen for recurrence of disease. (u) C. Transrectal ultrasound is mainly used for staging of disease and not monitoring of recurrence. (u) D. Radionuclide bone scan is used to detect bony metastases. Ref: (28) 35. Diagnosis/Dermatology A patient presents with profound itching. Examination reveals short, reddish lesions on the wrists, elbows, and finger webs. Papules are also noted in these areas. There appears to be burrow marks emanating proximal to the finger webs. Which of the following is the most likely diagnosis? A. Scabies B. Body lice C. Rocky Mountain Spotted Fever D. Lyme Disease Explanations (c) A. Scabies is an infestation of a mite that is usually spread by skin to skin contact. Patients present with intractable pruritus, often with minimal cutaneous findings. There may be an associated inflammatory papule or nodule along with a burrow that is sometimes seen early in the course of the infestation. (u) B. Body lice or pediculosis pubis is an infestation of hair-bearing regions, most commonly in the pubic area but may also be seen on hairy parts of the chest, axillae, and upper eyelashes. It is characterized by mild to moderate pruritus, papular urticaria, and excoriation. (u) C. Patients develop Rocky Mountain Spotted Fever after a tick bite. It is classically associated with a sudden onset of fever, severe headache, myalgia, and a characteristic acral exanthem. (u) D. Lyme disease is associated with an oval shaped skin lesion with central clearing known as erythema chronicum migrans. It occurs following a tick bite and may be associated with Bell's palsy, arthralgias, and heart block. Ref: (10) 36. Clinical Intervention/Endocrinology Which of the following is the most accurate method to differentiate benign from malignant thyroid nodules? A. Fine needle aspiration biopsy B. Ultrasound of the thyroid gland C. Measurement of thyroid antibodies D. Radionuclide iodine uptake Explanations (c) A. Fine needle aspiration biopsy is the best method used in the evaluation of thyroid cancer. Fine needle aspiration biopsy can be done on an outpatient basis and can be done under ultrasound guidance for patients who have small thyroid nodules. (u) B. Ultrasound of the thyroid gland is performed in order to help to guide the practitioner in performing a fine needle aspiration biopsy. It can also be used to differentiate solid from cystic masses but it cannot predict which nodules are cancer and which are benign. (u) C. Measurement of thyroid antibodies is most helpful in patients who have subclinical hypothyroidism or overt hyperthyroidism. There is a lot of controversy regarding whether treatment should be given to patients with subclinical disease and most providers will use thyroid antibodies to determine whether treatment should be given or delayed. (u) D. Radionuclide iodine uptake is performed to assess the activity of the thyroid gland. This test is most helpful to determine whether the gland is metabolically active (as seen with Graves' disease) or whether the gland is just releasing preformed thyroid hormone (subacute thyroiditis). The test provides a percentage of activity rather than a picture (thyroid scan) provides a picture of whether a nodule is "hot" or cold." Ref: (28)

20

37. Clinical Therapeutics/Cardiology A 16 year-old male with a history of tetralogy of Fallot presents to clinic for a follow-up visit status post replacement of his right ventricle to pulmonary artery conduit. He has complaints of chest pain with inspiration, fever and general malaise. Cardiac examination reveals a rub with muffled heart sounds. Labs show an elevated erythrocyte sedimentation rate (ESR) and leukocytosis. Which of the following is the most effective treatment? A. Acetaminophen/oxycodone B. Azithromycin C. Indomethacin D. Furosemide Explanations (u) A. See C for explanation. (u) B. See C for explanation. (c) C. Indomethacin is suitable for controlling pain in Dressler's syndrome. ASA is preferred. Narcotics, diuretics or antibiotics are not recommended. (u) D. See C for explanation. Ref: (11) 38. Diagnostic Studies/Gastrointestinal/Nutritional Which of the following laboratory tests, if positive, would be most indicative of Crohn's disease? A. Antineutrophil cytoplasmic antibodies (ANCA) B. Antiendomysial antibodies (AEA) C. Antinuclear antibodies (ANA) D. Anti-Saccharomyces cerevisiae antibodies (ASCA) Explanations (u) A. Antineutrophil cytoplasmic antibodies is positive in 60-70% of patients with ulcerative colitis while only 5-10% of Crohn's disease patients have positive results. (u) B. The antiendomysial IgA antibodies are directed against tissue transglutaminase and has a 90-95% specificity for celiac sprue. (u) C. Antinuclear antibodies are elevated in a wide variety of autoimmune disorders and therefore are a nonspecific finding. (c) D. About 60-70% of patients with Crohn's disease are positive for these antibodies which are directed at the cell walls of S. cerevisiae while only 10-15% of ulcerative colitis is found to be positive. Ref: (7)

39. Scientific Concepts/Orthopedics/Rheumatology A 22 year-old female complains of worsening pain, swelling, and tenderness in her left heel for 1 week. She sustained a penetrating injury to the heel two weeks ago when she stepped on a nail while running in tennis shoes. Examination reveals a draining puncture wound with surrounding erythema and exquisite tenderness. X-ray of the left foot demonstrates periosteal reaction associated with the wound. Which organism is classically responsible for this infection? A. Escherichia coli B. Streptococcus pyogenes C. Pseudomonas aeruginosa D. Staphylococcus aureus Explanations (u) A. While gram negative enteric organisms can be a cause of osteomyelitis, E. coli is rarely seen outside of the neonatal period. (u) B. Streptococcus pyogenes is an uncommon (~10 %) cause of acute osteomyelitis. The mechanism of injury suggests a different pathogen. (c) C. Pseudomonas aeruginosa is frequently associated with osteomyelitis involving puncture wounds of the foot. This is believed to result from direct inoculation with P. aeruginosa via the foam padding found in tennis shoes. (u) D. Staphylococcus aureus is the most common infecting organism in cases of acute osteomyelitis, however, the mechanism of injury in this case suggests P. aeruginosa. Ref: (5)

21

40. History & Physical/Hematology What examination finding would be expected in a patient with von Willebrand disease? A. Gingival bleeding B. Splenomegaly C. Muscle weakness D. Hemarthrosis Explanations (c) A. A patient with von Willebrand disease most commonly presents with mucosal bleeding seen in epistaxis, gingival bleeding, and menorrhagia. (u) B. Splenomegaly is seen with hemolytic anemia, not with von Willebrand disease. (u) C. Muscle weakness is not seen with von Willebrand disease. (u) D. Hemarthrosis is found in hemophilia, not in von Willebrand disease. Ref: (28) 41. Diagnosis/Psychiatry/Behavioral Medicine A 63 year-old retired engineer presents with one month of difficulty sleeping. He has a hard time staying asleep and says he is just restless. He also states that he has been more forgetful and can't pay attention very well. He lives with his wife. He denies fever, chills, recent trauma, or difficulty walking. When asked about the specifics of his symptoms, he repeatedly replies, "I don't know," without really trying. He states his wife thinks he just sits around the house all day since he retired. His neurological exam is unremarkable, except for some mild psychomotor retardation. Which of the following is the most likely diagnosis? A. Dementia B. Delirium C. Depression D. Dissociative disorder Explanations (u) A. Dementia is a deterioration of selective mental functions including progressive loss of impulse control and an attempt to cover up mental deficiencies. (u) B. Delirium has a rapid onset of symptoms, mental status fluctuations. Anxiety and irritability are common. (c) C. Depression often presents with difficulty thinking and concentrating, lessened sleep, and withdrawal from activities. (u) D. Dissociative disorder is associated with inability to recall important personal information usually of traumatic nature that is too extensive to be explained by ordinary forgetfulness.

Ref: (28) 42. Health Maintenance/Orthopedics/Rheumatology Which of the following is an established risk factor for osteoporosis? A. Parity status B. Carbohydrate intake C. Lactation history D. Low body weight Explanations (u) A. Parity status and lactation history have been shown to be poor predictors of bone mass; therefore, they do not have an established role in the prediction of increased risk for development of osteoporosis. (u) B. Carbohydrate intake has no association to the development of osteoporosis. (u) C. See A for explanation. (c) D. Established risk factors for osteoporosis include low body weight, female sex, advanced age, Caucasian race, and bilateral oophorectomy before menopause without estrogen replacement. Ref: (19)

22

43. Clinical Intervention/Obstetrics/Gynecology A 52 year-old obese patient with persistent heavy menses undergoes an endometrial biopsy and is diagnosed with atypical adenomatous hyperplasia. What is the next step in the management of this patient? A. Total abdominal hysterectomy B. Observation and endometrial biopsy in 3 months C. Endometrial curettage followed by progesterone daily D. Oral progesterone days 16-25 of the month for 6 months and repeat biopsy Explanations (c) A. Atypical adenomatous hyperplasia contains cellular atypia and mitotic figures in addition to glandular crowding and complexity. This has a 20-30% risk of progression to endometrial cancer and the recommendation is hysterectomy. (h) B. Observation and biopsy again in 3 months would increase the risk of endometrial cancer for this patient. (h) C. Endometrial curettage would remove the hyperplasia and progesterone will decrease the endometrial glandular proliferation. This would be appropriate management in a patient with endometrial hyperplasia without atypia. (h) D. Oral progesterone for 10 days of the month will cause the patient to have a withdrawal bleed every month. This would be an appropriate treatment in a premenopausal patient with endometrial hyperplasia without atypia Ref: (4) 44. Diagnostic Studies/Cardiology A 72 year-old male presents to the emergency department with crushing chest pain, dyspnea and palpitations for 2 hours in duration. Enzymes are pending and he has been given aspirin and sublingual nitroglycerin. He is rushed to the catheterization lab where they find a totally occluded distal right coronary artery. Which of the following electrocardiogram (ECG) findings supports the diagnosis?

A. Q waves in leads I, aVL, V5-V6 B. ST segment elevation in leads II, III, aVF C. Hyperacute T waves in leads I, aVL D. Flipped T waves with repolarization changes in leads V1-V4 Explanations (u) A. Q waves in leads I, aVL, V5-V6 represent infarction involving the circumflex artery. (c) B. ST segment elevation in leads II, III, aVF, represents an acute process in the right coronary artery. (u) C. Hyperacute T waves in leads I, aVL can represent the initial changes of an infarction involving the circumflex artery. (u) D. Flipped T waves with repolarization changes in leads V1-V4 can represent early stages of infarction involving the left anterior descending artery. Ref: (11) 45. Clinical Therapeutics/Pulmonology A patient presents with occasional wheezing and chest tightness that occurs approximately once a week and at night only about once a month. Peak expiratory flow is 85% of predicted. Which of the following is the most appropriate initial treatment? A. Albuterol (Proventil) inhaler B. Montelukast (Singular) C. Salmeterol (Serevent) inhaler D. Sustained release theophylline Explanations (c) A. This patient has mild intermittent asthma which is initially treated with inhaled beta 2-agonists as needed. No long-term control medications are indicated. (u) B. Leukotriene modifiers, such as montelukast, may be added to the treatment of uncontrolled asthma as a long-term controller after the initiation of inhaled corticosteroids. (u) C. Long-acting beta 2-agonists, such as salmeterol, are indicated for long-term control of asthma that is categorized as moderate persistent to severe persistent. (u) D. Sustained release theophylline is an alternative treatment for asthma that is at least categorized as mild persistent; however its narrow therapeutic window and side effects limit its use. Ref: (28)

23

46. History & Physical/Urology/Renal Which of the following signs and symptoms is typically noted in patients with acute cystitis? A. Fever and chills B. CVA tenderness C. Flank pain D. Frequency and dysuria Explanations (u) A. Signs of systemic toxicity, such as fever and chills, are absent in acute cystitis. (u) B. CVA tenderness and flank pain are associated with acute pyelonephritis. (u) C. See B for explanation. (c) D. Irritative voiding symptoms, such as frequency and dysuria, are common in acute cystitis. Ref: (28) 47. Diagnosis/Endocrinology A 54 year-old female presents to the office for radiographic and laboratory results. The radioactive iodine uptake is elevated while the thyroid hormone levels are increased with TSH levels being suppressed. Which of the following is the most likely diagnosis? A. Graves' disease B. Hashimoto's thyroiditis C. Subacute thyroiditis D. Pituitary failure Explanations (c) A. Graves' disease is associated with an elevated uptake on the radioactive thyroid scan due to an increase in the activity for the thyroid gland. Because the gland is actually making thyroid hormone, the free T4 level will be increased and the TSH will be suppressed as a result of negative feedback to the hypothalamus and pituitary gland. (u) B. Patients with Hashimoto's thyroiditis will have an underactive thyroid gland with a decrease in the radioactive iodine uptake. The thyroid gland is not producing enough thyroid hormones so the free T4 level will be low and the TSH level will be increased as the body tries to stimulate the thyroid gland to produce more thyroid hormone. (u) C. Patients with subacute thyroiditis have an increase in the release of thyroid hormone rather than an increase in the actual activity of the gland. These patients will have a normal or decreased iodine uptake on the radioactive thyroid scan which is the main differentiating feature between this condition and Graves' disease. The free T4 level can be variable and depends upon when in the course of this condition the thyroid hormones are measured. (u) D. Patients with pituitary failure will not be able to produce thyroid stimulating hormone so these patients will have a decreased radioactive thyroid uptake, a low free T4 level and a low TSH. Ref: (28) 48. Scientific Concepts/Neurology The source of pain experienced during a migraine headache is a result of activation of which nerve? A. Trigeminal B. Vagus C. Optic D. Occulomotor Explanations (c) A. Headache may result in release of neuropeptides acting as neurotransmitters at trigeminal nerve branches. (u) B. See A for explanation. (u) C. See A for explanation. (u) D. See A for explanation. Ref: (28)

24

49. Clinical Intervention/ENT/Ophthalmology A 17 year-old male is accidentally struck in the right eye while playing football and is immediately transported to the hospital. In the emergency room, he complains of severe pain behind the eye as well as double vision. On examination, he has exophthalmos, cannot move his right eye upward and blood is noted in the anterior chamber. Which of the following is the most appropriate course of action at this time? A. Apply ice packs and cold compresses B. Immediately refer the patient to an ophthalmologist C. Attempt to keep the patient calm and order a skull x-ray D. Administer a dose of intramuscular broad-spectrum antibiotic Explanations (u) A. See B for explanation. (c) B. This scenario describes a "blow-out" fracture of the orbit with hyphema and, because of the signs and symptoms presented, warrants an immediate consult by an ophthalmologist. (u) C. See B for explanation. (u) D. See B for explanation. Ref: (29) 50. Health Maintenance/Cardiology Which of the following population groups represent the greatest risk for developing primary hypertension? A. White non-Hispanic B. Hispanic C. Mediterranean D. Black non-Hispanic Explanations (u) A. White non-Hispanic adults have a low risk of hypertension compared to Hispanic and Black non-Hispanics. (u) B. Hispanic adults are lower risk of hypertension than Black non-Hispanic, but not compared to White non-Hispanic. (u) C. Mediterranean adults have a lower risk of hypertension than Black non-Hispanics. (c) D. Black non-Hispanic adults have the highest risk of hypertension. Ref: (28) 51. Clinical Therapeutics/Gastrointestinal/Nutritional In a patient with metabolic acidosis due to ingestion of methanol, which substance below is considered therapeutic? A. Salicylates B. Paraldehyde C. Ethylene glycol D. Ethanol Explanations (u) A. Salicylates, paraldehyde, ethylene glycol all cause metabolic acidosis with an elevated anion gap. (u) B. See A for explanation. (u) C. See A for explanation. (c) D. Ethanol, along with hemodialysis and supportive measures, is indicated for metabolic acidosis caused by methanol ingestion. Ref: (28) 52. History & Physical/Dermatology Which of the following is the only disease that forms an ulcer at the site of inoculation? A. Tularemia B. Scabies C. Lyme disease D. Rocky Mountain Spotted Fever

25

Explanations (c) A. Tularemia is an acute infection that is transmitted by handling the flesh of infected animals, by the bites of insect vectors and by inoculation of conjunctiva. (u) B. Scabies is caused by the infestation of a mite and is associated with significant pruritus. There may be minimal cutaneous findings although a burrow and skin tracks may be seen early in the course of this disease. (u) C. Lyme disease is associated with a rash known as erythema chronicum migrans. The rash has an expanding border and central clearing. (u) D. Systemic symptoms predominate with Rocky Mountain Spotted Fever and the rash has an acral distribution pattern that may occur on the palms and soles. It does not have any ulcer associated with it. Ref: (10) 53. Diagnosis/Pulmonology A 4 year-old patient presents with episodic wheezing and a non-productive cough for the last 4 weeks. His symptoms are worse at night. Past medical history reveals a history of atopic dermatitis. Physical examination at this time is unremarkable. Which of the following is the most likely diagnosis? A. Asthma B. Bronchiolitis C. Croup D. Cystic fibrosis Explanations (c) A. Asthma is a chronic inflammatory disorder of the airways. It is characterized by episodic or chronic symptoms of airflow obstruction, breathlessness, cough, wheezing, and chest tightness. The strongest identifiable predisposing factor for the development of asthma is atopy. (u) B. Bronchiolitis is common in infants and young children presenting with acute onset of cough, rhinorrhea, tachypnea, and expiratory wheezes. (u) C. Croup usually presents with a prodrome of upper respiratory tract symptoms followed by onset of a barking cough and stridor. (u) D. Cystic fibrosis is an autosomal recessive disease and is characterized by a chronic cough, sputum production, dyspnea, and wheezing. Steatorrhea, diarrhea, and abdominal pain are also common. Ref: (13) 54. Diagnostic Studies/Obstetrics/Gynecology A 23 year-old female is in active labor and has progressed from 3 cm to 6 cm in the last six hours. Fetal monitoring demonstrates mild repetitive late decelerations. Which of the following is the most likely cause of this finding? A. Fetal hypoxia B. Head compression C. Cord compression D. Uteroplacental insufficiency Explanations (u) A. Fetal hypoxia would be a concern if deep late FHR decelerations were present with absent beat-to-beat variability. (u) B. Early decelerations are due to head compression of the fetus. Pressure on the fetal head causes an alteration in cerebral blood flow causing a central vagal stimulation and subsequent FHR deceleration. The deceleration is a mirror image of the contraction. (u) C. Variable decelerations are from cord compression. The decelerations have a sharp, angular, decline in FHR with duration less than 2 minutes. (c) D. Late decelerations are from uteroplacental insufficiency. The decelerations have a smooth, gradual symmetrical decrease in FHR beginning at or after the peak of the contraction. Ref: (4) 55. Clinical Intervention/Orthopedics/Rheumatology A 60 year-old female injured her right wrist when she slipped and fell onto her outstretched hand. Radiographs show a fracture through the metaphysis of the distal radius with dorsal displacement and angulation. Which of the following splints is the best method of temporary immobilization?

26

A. Dorsal forearm B. Ulnar gutter C. Volar forearm D. Volar with thumb spica Explanations (u) A. The dorsal forearm splint is best used as an alternative to the ulnar or radial gutter splint for protection of fractures of metacarpals two through five. (u) B. Ulnar gutter splints are best for treatment of stable fractures and dislocations of the ulnar wrist and metacarpals. (c) C. The volar forearm splint is best for temporary immobilization of forearm, wrist and hand fractures and is the splint of choice for Colles' fracture. (u) D. A volar splint with thumb spica is used to immobilize the first metacarpophalangeal joint and is useful for scaphoid fractures. Ref: (27) 56. Clinical Therapeutics/Psychiatry/Behavioral Medicine A 36 year-old patient presents requesting something to help him sleep. He reports that he has always had a problem sleeping, admits to feeling nervous most days for the last 2 years, and that he has always been "uptight" and a "worry wart." During the previous eight months he has frequently felt tense, shaky, sweaty, with palpitations and frequent headaches. He reports being irritable with his 5 year-old son. Which of the following is the best treatment option for this patient? A. Alprazolam (Xanax) B. Haloperidol (Haldol) C. Paroxetine (Paxil) D. Diphenhydramine (Benadryl)

Explanations (u) A. Benzodiazapines should not be used in generalized anxiety disorder (GAD) as they only help symptoms short-term and should not be used long-term. (u) B. Antipsychotic medications are not useful in generalized anxiety disorder. (c) C. SSRI's, specifically Paxil, are the mainstay for treatment of generalized anxiety disorder. (u) D. Antihistamines are not useful in the treatment of generalized anxiety disorder. Ref: (28) 57. Diagnosis/Cardiology A 60 year-old male with hypertension is brought to the emergency department 30 minutes after the sudden onset of severe chest pain that radiates to his back and arms. His blood pressure is 180/80 mmHg in his left arm; no blood pressure reading can be obtained from the right arm. ECG shows sinus tachycardia with left ventricular hypertrophy. A high pitched decrescendo diastolic murmur is heard along the left mid-sternal border. Which of the following is the most likely diagnosis? A. Acute myocardial infarction B. Aortic dissection C. Pulmonary embolism D. Right subclavian arterial embolus Explanations (u) A. Although included as part of the differential the presentation is not consistent with AMI. ECG may show ST changes and a murmur of mitral regurgitation may be present with papillary muscle rupture. (c) B. This is a classic presentation for aortic dissection. (u) C. Patients will also present with shortness of breath, feelings of impending doom and chest pain that varies with respirations. (u) D. Arterial embolus will present with symptoms related to the location of the occlusion. Pain and paresthesias are usually the earliest symptoms. Ref: (28)

27

58. Health Maintenance/Dermatology Which of the following patients is at highest risk for the development of the skin disorder known as erythrasma? A. Lactating women B. Postmenopausal women C. Seniors living in cold northern climate D. People living in tropical climate Explanations (u) A. Lactating women are at increased risk for staph infections in the breast. (u) B. Postmenopausal women are at increased risk for atrophic vaginitis but not for erythrasma. (u) C. Seniors living in cold conditions are at risk for winter itch which is worse with dry heat. (c) D. People living in warm, tropical climate, people wearing occlusive clothing or shoes, obese patients, and those with hyperhidrosis are at increased risk for erythrasma. The diagnosis of this condition is made by demonstration of a coral red fluorescence. Ref: (10) 59. History & Physical/Gastrointestinal/Nutritional What is the term for blue discoloration about the umbilicus? A. Cullen's sign B. Murphy's sign C. Rovsing's sign D. Turner sign Explanations (c) A. Cullen's sign is a blue discoloration about the umbilicus and can occur in hemorrhagic pancreatitis and results from hemoperitoneum. (u) B. Murphy's sign is seen in liver and gallbladder disease by which the patient abruptly halts deep inspiration due to pain in the right upper quadrant while the examiner's hand is keeping stable pressure to the area. (u) C. Rovsing's sign is positive when the patient experiences right sided abdominal pain with palpation to the left side. This is generally seen in those with appendicitis. (u) D. Turner sign is also seen in severe, acute pancreatitis but is represented by a green-brown discoloration of bilateral flanks. Ref: (29) 60. Clinical Therapeutics/ENT/Ophthalmology A 22 year-old woman presents with sneezing, runny nose, postnasal drip, and nasal congestion for the last week. She says this happens every spring. She is not allergic to any medications. Which of the following is the most appropriate pharmacologic treatment for this patient? A. Azithromycin (Zithromax) B. Phenylephrine (Neo-synephrine) C. Nedocromil D. Pseudoephedrine Explanations (u) A. Azithromycin is used to treat bacterial infections not allergic disorders. (u) B. Neo-synephrine is not indicated in the treatment of allergic rhinitis. (c) C. Nedocromil inhibits mast cell degranulation and is an effective treatment for allergic rhinitis. It may take 2-6 weeks for full therapeutic effect. (u) D. Pseudoephedrine is a decongestant that may relieve the nasal congestion, but has no effect on the allergic response. Ref: (28)

28

61. Diagnostic Studies/Urology/Renal A 60 year-old patient presents with elevated blood pressure and peripheral edema. Laboratory testing reveals a BUN of 58 mg/dl and a creatinine of 4.5 mg/dl, these results are unchanged from six months ago. Urinalysis today is negative except for the following, specific gravity of 1.002, 2+ protein, and trace glucose. Which of the following laboratory findings would be most consistent for this patient? A. Hypercalcemia B. Metabolic alkalosis C. Hypokalemia D. Anemia Explanations (u) A. In a patient with chronic renal failure, typical laboratory findings include hypocalcemia, metabolic acidosis, and hyperkalemia. (u) B. See A for explanation. (u) C. See A for explanation. (c) D. Anemia, due to low erythropoietin, is common in patients with chronic renal failure. Ref: (11) 62. Scientific Concepts/Pulmonology Which of the following types of pleural effusion result from increased production of fluid due to underlying inflammatory conditions? A. Exudate B. Empyema C. Transudate D. Hemothorax Explanations (c) A. Increased production of fluid due to inflammatory or malignant processes results in an exudative pleural effusion. (u) B. Infection in the pleural space causes an empyema. (u) C. Transudates result from increased hydrostatic or decreased oncotic pressures across normal capillaries. (u) D. Bleeding into the pleural space causes a hemothorax. Ref: (28) 63. History & Physical/Cardiology Which of the following conditions is most suggestive of an abdominal aortic aneurysm? A. Abdominal mass B. Hypertension C. Chest pain D. Syncope Explanations (c) A. An abdominal aortic aneurysm presents with a pulsatile upper abdominal mass. (u) B. Hypertension is not suggestive of an abdominal aortic aneurysm. (u) C. Abdominal aortic aneurysm presents with midabdominal or lower back pain. (u) D. Syncope is not common in abdominal aortic aneurysm, unless it ruptures. Ref: (7) 64. Diagnosis/Obstetrics/Gynecology A 16 year-old G0P0 patient presents complaining of lower pelvic pain that alternates from right to left side of her pelvis. She states that it is related to her cycle and occurs most commonly midcycle. She denies sexual activity. She reports that she has taken ibuprofen at the time of the discomfort with some relief. Her pelvic examination is unremarkable. Which of the following is the most likely diagnosis? A. Endometriosis B. Mittelschmerz C. Functional ovarian cyst D. Pelvic inflammatory disease

29

Explanations (u) A. With endometriosis, the uterus is often fixed and retroflexed in the pelvis. The palpable mass is an endometrioma or "chocolate cyst". The patient with endometriosis also often has dysmenorrhea, dyspareunia, and dyschezia. (c) B. Women may experience pain at the time of ovulation, may alternate side to side. (u) C. Functional ovarian cysts occur from ovulation and are not usually symptomatic. (u) D. Patients with pelvic inflammatory disease often present with fever, pain, and more acute symptoms. Ref: (4) 65. Clinical Intervention/Neurology A 67 year-old female with history of hypertension, diabetes mellitus, and smoking presents to the emergency department with mild expressive aphasia, right facial weakness and mild right arm weakness. She had awakened 60 minutes ago and was speaking to her husband when her speech suddenly became difficult to understand and weakness was noted. Physical examination reveals a blood pressure of 165/85 mm Hg. A CT of the head shows no intracranial hemorrhage. Which of the following is the most appropriate intervention? A. Aspirin B. Warfarin (Coumadin) C. Tissue plasminogen activator (rt-PA) D. Clopidogrel (Plavix) Explanations (u) A. Aspirin is used for preventative purposes and will not resolve a current embolism. (u) B. Warfarin is a preventative medicine that will not help the current situation. (c) C. IV thrombolytic therapy with recombinant tissue plasminogen activator is effective in reducing the neurological deficit in selected patient without CT evidence of intracranial hemorrhage and when administered within 3 hours after onset of ischemic stroke. (u) D. Plavix is a platelet aggregation inhibitor and used for preventative measures. Ref: (28) 66. Clinical Therapeutics/Gastrointestinal/Nutritional Which of the following medications decreases the exchange of hydrogen for potassium by inhibiting hydrogen, potassium-ATPase? A. Ranitidine (Zantac) B. Misoprostol (Cytotec) C. Sucralfate (Carafate) D. Omeprazole (Prilosec) Explanations (u) A. Ranitidine is an H2 receptor antagonist. (u) B. Misoprostol is a prostaglandin analogue. (u) C. Sucralfate is a coating agent. (c) D. Omeprazole, a proton pump inhibitor, works by inhibiting hydrogen, potassium-ATPase. Ref: (15) 67. Diagnosis/Orthopedics/Rheumatology A 32 year-old male presents with an acute onset of pain and swelling to his left ankle. On physical examination the ankle is warm, swollen and erythematous. Evaluation of the synovial fluid reveals only leukocytosis with a low glucose. Which of the following is the most likely diagnosis? A. Gout B. Pseudogout C. Acute rheumatic fever D. Septic arthritis Explanations (u) A. Gout and pseudogout are excluded by the failure to find crystals on synovial fluid analysis. (u) B. See A for explanation. (u) C. Acute rheumatic fever commonly involves multiple joints. (c) D. Leukocytosis and a low synovial glucose are indicative of septic arthritis. Ref: (28)

30

68. History & Physical/Endocrinology Which of the following clinical signs or symptoms is most likely to be present following iatrogenic injury during subtotal thyroidectomy? A. Inability to shrug the shoulders B. Hoarseness C. Impaired gag reflex D. Impaired taste sensation anterior two-thirds of tongue Explanations (u) A. Patients with injury to the spinal accessory (Cranial Nerve XI) will be unable to shrug the shoulders against resistance. This nerve is most likely to be injured during carotid endarterectomy and not subtotal thyroidectomy. (c) B. The recurrent laryngeal nerve is most commonly injured during subtotal thyroidectomy. If injured, hoarseness is the most common presentation for this nerve impairment. (u) C. Injury to the glossopharyngeal and vagus nerves are most likely to result in impaired gag reflex. These nerves are most likely to be injured during carotid endarterectomy and not during subtotal thyroidectomy. (u) D. The facial nerve (Cranial Nerve VII) provides taste to the anterior two-thirds of the tongue. It is not at risk for injury during subtotal thyroidectomy. Ref: (3) 69. Diagnostic Studies/Infectious Diseases A patient states that he has been camping in the mountains of North Carolina for the past two weeks. He presents to the clinic complaining of "flu-like" symptoms for the past 10-14 days however he notes that he started to develop a slight rash on his wrist and ankles about seven days ago. Which of the following tests would provide a confirmatory diagnosis? A. Immunofluorescent assay B. C-reactive protein C. Heterophile agglutination D. Anti-streptolysin O titer

Explanations (c) A. The immunofluorescent assay will confirm antibodies to Rickettsia. (u) B. C-reactive protein is a protein found in patients with inflammation during the acute phase illness. It is non-specific. (u) C. Heterophile agglutination is used to detect antibodies to the Epstein-Barr virus which is responsible for infectious mononucleosis. (u) D. ASO titer is done to detect antibodies to streptococcal infection. Ref: (28) 70. Health Maintenance/Cardiology Who is most likely to require subacute bacterial endocarditis (SBE) prophylaxis prior to a dental procedure? A. 22 year-old female with mitral valve prolapse B. 36 year-old male with a bio-prosthesic mitral valve C. 45 year-old female with an ASD closure 8 months ago with no residual defect D. 15 year-old male with a bicuspid aortic valve Explanations (u) A. See B for explanation. (c) B. The AHA recommends that patients with prosthetic heart valves receive antibiotic prophylaxis. As should cardiac transplant recipients with valve disease, unrepaired cyanotic CHD, repaired CHD with prosthetic material or device during the first six months of the procedure and repaired CHD with residual defects at site of patch or prosthetic device. (u) C. See B for explanation. (u) D. See B for explanation. Ref: (28)

31

71. Clinical Intervention/Pulmonology A 3 month-old male presents with two days of worsening hoarse cough and thick purulent rhinorrhea associated with increasing problems breathing and trouble feeding. Examination reveals a temperature of 100.2 degrees F and respiratory rate of 80/minute with nasal flaring and retractions. Lung examination reveals a prolonged expiratory phase with inspiratory rales. He is tachycardic. Pulse oximetry reveals oxygen saturation of 89%. Chest x-ray reveals hyperinflation with diffuse interstitial infiltrates. Which of the following is the most appropriate intervention? A. Antibiotics B. Hospitalization C. Inhaled corticosteroids D. Racemic epinephrine Explanations (u) A. Antibiotics are utilized to treat bacterial, not viral, illnesses. (c) B. This infant most likely has bronchiolitis. While most cases are mild and can be treated at home, hospitalization is recommended for infants with hypoxia on room air, moderate tachypnea with feeding difficulties and marked respiratory distress with retractions. Additionally hospitalization is recommended for infants less than 2-3 months of age, a history of apnea or an underlying chronic cardiopulmonary disease. (u) C. The use of corticosteroids in children with bronchiolitis has not been studied and does not appear to be helpful. (u) D. Racemic epinephrine is not indicated in the treatment of bronchiolitis. Ref: (14) 72. Clinical Therapeutics/Obstetrics/Gynecology A patient with preterm labor may be given corticosteroids to A. decrease uterine activity. B. prevent chorioamnionitis. C. enhance fetal lung maturity. D. prevent the development of gestational diabetes. Explanations (u) A. Tocolytics are given to decrease uterine activity with preterm labor. (u) B. IV antibiotics are given to patients with chorioamnionitis. (c) C. Corticosteroids may be given from 24-34 weeks in patients with preterm labor or who have pregnancy complications which may cause premature birth. The corticosteroids enhance pulmonary maturity. (u) D. There are no medications to prevent the development of gestational diabetes, however, patients who have gestational diabetes may be treated with a diabetic diet and/or insulin to manage the condition and decrease complications. Ref: (4) 73. History & Physical/ENT/Ophthalmology Which of the following would indicate an optic nerve lesion?

A. Excessive conjunctival edema B. Ptosis C. Inability to gaze laterally D. Afferent pupillary defect Explanations (u) A. Excessive edema of the conjunctiva is a feature of chemosis. (u) B. Ptosis is not indicative of an optic nerve lesion. (u) C. Inability to gaze laterally would be due to paralysis of the lateral rectus muscle controlled by cranial nerve VI. (c) D. Pupil size, controlled centrally by the Edinger-Westphal nucleus in the midbrain, is primarily based on the afferent light stimulus transmitted via the optic nerve. Ref: (3)

32

74. Scientific Concepts/Cardiology When evaluating jugular venous pulsations a prominent a wave represents which of the following? A. Atrial contraction against a closed tricuspid valve B. Rapid filling of the right atrium C. Tricuspid regurgitation D. Poor left ventricle compliance Explanations (c) A. The a wave corresponds to right atrial contraction. (u) B. See A for explanation. (u) C. See A for explanation. (u) D. See A for explanation. Ref: (11) 75. Diagnosis/Psychiatry/Behavioral Medicine A patient presents with an episode of an expansive, elevated mood during which she cleaned excessively without sleeping. Which of the following is the most likely diagnosis?

A. Major depressive disorder B. Bipolar disorder C. Schizoaffective disorder D. Dysthymic disorder Explanations (u) A. Criteria for major depressive disorder do not include elevated expansive moods. (c) B. Bipolar disorder is characterized by episodic mood shifts from depression to manic type moods which is often rapid with depression lasting longer than manic episodes. Bipolar disorder may initially present with a manic episode. (u) C. Schizoaffective disorder presents with a mood disorder and characteristics of schizophrenia. (u) D. Dysthymia has no elevated moods or manic type behaviors. Ref: (28) 76. Clinical Intervention/Urology/Renal A 27 year-old patient presents with crush injuries to both lower extremities after being involved in an automobile accident. Within six hours of the accident, urine output has decreased to less than 10 cc per hour. Within 24 hours the serum creatinine increased from 0.9 mg/dl to 2.9 mg/dl and serum CPK is now 12,000 U/L. In addition to a fluid challenge with normal saline, which of the following is the most appropriate treatment as this time? A. IV calcium chloride B. Oral captopril (Capoten) C. IV sodium bicarbonate D. Oral sevelamer (Renagel) Explanations (u) A. While hypocalcemia is common in rhabdomyolysis and acute renal failure, treatment is not needed unless clinical signs such as tetany are noted. (h) B. The use of ACE inhibitors, such as captopril, may worsen renal function and are not indicated in this case. (c) C. Treatment of acute renal failure due to rhabdomyolysis is best accomplished with IV fluids and forced alkaline diuresis. (u) D. Sevelamer is a phosphate binder used to treat elevated phosphate levels in patients with end stage renal disease. Ref: (11)

33

77. Clinical Therapeutics/Neurology A 38 year-old female has a 10 year diagnosis of migraine headaches. She had been using ergotamine to abort her headaches, but is now having one or two headaches per week that are interfering with work. Which of the following is the most appropriate preventive therapy? A. Sumatriptan (Imitrex) B. Promethazine (Phenergan) C. Propranolol (Inderal) D. Ketorolac (Toradol) Explanations

(u) A. Imitrex is not approved for preventive therapy but is used as abortive therapy. (u) B. Phenergan is an antiemetic that helps with nausea and vomiting for people who get migraines. It is not a preventive medication. (c) C. Propanolol is one beta blocker that is frequently used as a first-line prophylaxis for migraines. (u) D. Ketorolac is not indicated for scheduled, daily use as a preventive for migraines. Ref: (28) 78. History & Physical/Cardiology An 18 year-old male high school basketball player comes to clinic for a routine physical exam. His height is 193 cm (76 in.); arm span is 201 cm (79 in.). He has long fingers and toes. Blood pressure is 146/62 mmHg and pulse is 64/min. Which of the following exam findings is most consistent with the diagnosis? A. Grade 2/6 high-frequency diastolic murmur at the third right intercostal space B. Grade 2/6 systolic ejection murmur at the second left intercostal space with a fixed widely split S2 C. Grade 2/6 continuous murmur heard best at the high left sternal border D. Grade 2/6 systolic murmur at the fourth left intercostal space that decreases with squatting Explanations (c) A. This murmur is most consistent with aortic regurgitation which can be present in patients with Marfans syndrome and a dilated aortic root. (u) B. This murmur is most consistent with an atrial septal defect. (u) C. This murmur is most consistent with a patent ductus arteriosus and unlikely in this age patient. (u) D. This murmur is most consistent with hypertrophic obstructive cardiomyopathy. Ref: (28) 79. Diagnostic Studies/Dermatology A patient is concerned about being exposed to condyloma acuminata. Which of the following tests is most appropriate to perform in order to better identify these lesions? A. Acetowhitening B. Tzanck smear C. Potassium hydroxide test D. Wood's light fluoroscopy Explanations (c) A. Acetowhitening is used to facilitate the diagnosis of condyloma acuminata lesions. A 3 to 5% acetic acid solution is applied to these suspected genital warts for five to ten minutes. Condyloma lesions will whiten and appear as circumscribed macular or papular lesions with a granular surface. (u) B. Tzanck smears are used in making the diagnosis of herpetic lesions, such as herpes simplex, varicella, and herpes zoster infection. (u) C. Potassium hydroxide tests are used to rule out the presence of dermatophyte (fungal and yeast) infections. (u) D. Wood's light fluoroscopy is used in the assessment of dermatophyte infection and a bacterial infection known as erythrasma. Ref: (2)

34

80. Health Maintenance/Pulmonology Which of the following is an indication for a pediatric patient to receive the 23-valent polysaccharide vaccine (Pneumovax)? A. Children at any age with a history of asthma B. All children at 2,4,6 and 12-18 months of age C. All children at 12-23 months of age in a two dose series D. Children age 24-59 months at high risk for invasive pneumococcal disease Explanations (u) A. Pediatric patients with cystic fibrosis, not asthma, are included in the indications for vaccination with Pneumovax, however they must be at least 2 years old. (u) B. The 7-valent pneumococcal conjugate vaccine (Prevnar) is currently recommended to be given to children under the age of two on the schedule outlined. (u) C. While a two dose series is recommended for appropriate pediatric patients that receive Pneumovax, the recommended timing between doses is 3-5 years. (c) D. Pneumovax is licensed for use in children over the age of 23 months and is indicated for all pediatric patients at increased risk for pneumococcal disease. Ref: (13) 81. Diagnosis/Endocrinology A patient presents to the office with worsening fatigue, weight loss, and weakness. She notes that she is having recurrent bouts of abdominal pain and has been losing her pubic hair. Patient is found to have orthostatic hypotension. Which of the following conditions is most likely? A. Cushing's syndrome B. Pheochromocytoma C. Primary hyperparathyroidism D. Addison's disease Explanations (u) A. Cushing's syndrome is caused by an increase in the cortisol levels in the body. These patients will have hypertension, buffalo hump, striae, and proximal muscle weakness. (u) B. Pheochromocytoma is caused by an increase in the release of catecholamines from the adrenal medulla. These patients will have episodic hypertension followed by sustained hypertension and bouts of diaphoresis and shakiness. (u) C. Patients with primary hyperparathyroidism are most likely to be asymptomatic. If these patients have symptoms, it is most likely that they will have abdominal pain, renal stones, and bone pain because of the resultant increase in the serum calcium levels. (c) D. Patients with Addison's disease have primary adrenal failure from an autoimmune problem in the adrenal gland or due to hemorrhage into the adrenal gland. These patients are not able to make glucocorticoids, mineralocorticoids, or sex hormones which result in hypotension, hyperpigmentation (from an increase in the ACTH and MSH hormones) and are hyponatremic. Ref: (28) 82. Clinical Intervention/Gastrointestinal/Nutritional A patient presents to the emergency department with right upper quadrant pain over eight hours, nausea, and vomiting. On exam there is a fever of 101.2 degrees F. Ultrasound shows a distended gallbladder. What is the most appropriate management of this patient? A. Oral analgesics B. Diagnostic peritoneal lavage C. Proton pump inhibitors D. Laparoscopic cholecystectomy Explanations (u) A. See D for explanation. (h) B. Diagnostic peritoneal lavage is used to detect intraabdominal bleeding from trauma and not to treat acute cholecystitis and may delay appropriate treatment. (u) C. Proton pump inhibitors are used to treat GERD or PUD. (c) D. The proper treatment for acute cholecystitis is IV fluids, antibiotics, pain control, and surgery. Cholecystectomy is the definitive treatment for acute cholecystitis and laparoscopic cholecystectomy is the procedure of choice. Ref: (25)

35

83. Clinical Therapeutics/Hematology A 56 year-old male is noted to have a recent diagnosis of polycythemia vera. His current hemoglobin is 21 gms/dl. What treatment should be instituted for this patient at this time? A. Phlebotomy B. Iron chelation therapy C. Bone marrow radiation therapy D. Normal saline IV hydration

Explanations (c) A. This patient has a diagnosis of polycythemia vera treatment begins with phlebotomy instituted on a weekly basis until the hematocrit is less than 45%. Maintainance of the hematocrit at 45% is achieved with repeated phlebotomy as necessary. (u) B. See A for explanation. (h) C. Radiation therapy has no place in this treatment. (h) D. IV hydration with normal saline will place the patient into volume overload. Ref: (28) 84. History & Physical/Obstetrics/Gynecology An 18 year-old female comes to the clinic with the complaint of increased vaginal discharge and vaginal odor. She also complains of urinary frequency. On physical examination there is evidence of thin, gray, frothy discharge in the vagina. The cervix appears erythematous and the vaginal pH is 6. Which of the following is the most likely diagnosis? A. Candida vaginitis B. Bacterial vaginosis C. Trichomonas vaginitis D. Chlamydia trachomatis Explanations (u) A. Patients with a yeast infection most often present with vulvar/vaginal pruritus, burning, dyspareunia "cottage-cheese" discharge which is odorless, pH is often normal 4-4.5. (u) B. Symptoms from bacterial vaginosis include ivory to gray discharge, thin, homogeneous, adherent, often increased pH 5-6.5 (basic), distinctive "fishy" odor, itching may be present. Malodorous discharge is especially noticeable by the patient after menses or intercourse. (c) C. Signs of trichomonas include: thin frothy or bubbly, pale yellow-green to gray adherent vaginal discharge, can have erythema of vulva and vagina, may have petechiae on the cervix, amine odor may be present, may also complain of dysuria and dyspareunia, pH 5 to 6.5 (basic). (u) D. Patients with Chlamydia are often asymptomatic but may have mucopurulent discharge and cervical inflammation. Ref: (4) 85. Diagnostic Studies/Orthopedics/Rheumatology An obese 15 year-old male presents with complaint of a limp and right knee pain for two weeks. He denies recent trauma or history of previous injury. Physical examination of the right knee is unremarkable. Examination of the right hip reveals pain with passive range of motion and limited internal rotation and abduction. Flexion of the hip results in external rotation of the thigh. Gait is antalgic with the right hip externally rotated. Which of the following radiographic findings supports the most likely diagnosis? A. Displacement of the femoral epiphysis B. Irregularity and fragmentation of the joint space C. Capsular swelling of the joint D. Dislocation of the hip Explanations (c) A. This patient has slipped capital femoral epiphysis (SCFE) and the classic x-ray findings will demonstrate displacement of the femoral head rotation of the femoral neck anteriorly. (u) B. Irregularity and fragmentation of the joint space is associated with avascular necrosis of the femoral head as seen in Legg-Calve-Perthes disease. This typically occurs in a younger male population and is not associated with the classic externally rotated hip with ambulation seen in SCFE. (u) C. Capsular swelling of the joint may be seen in transient synovitis of the hip but is not associated with SCFE. (u) D. Hip dislocation at this age is associated with major trauma, such as that sustained in a fall from height or dashboard injury. SCFE does not lead to hip dislocation. Ref: (5)

36

86. Scientific Concepts/Cardiology Which of the following is the most likely initial effect on the left ventricle from aortic stenosis? A. Dilitation of the ventricle with diastolic dysfunction B. Wall stiffness due to ischemia from decreased coronary blood flow C. Paradoxical wall motion abnormalities due to increased preload D. Concentric hypertrophy with preserved function Explanations (u) A. Dilitation of the ventricle is a later finding. (u) B. This is not an effect from aortic stenosis but coronary artery blockage. (u) C. Preload is the end-diastolic pressure or volume within the ventricle, ischemic heart disease or infarct would cause paradoxical wall motion abnormalities due to the increased preload. (c) D. Hypertrophy would be the initial changes of the left ventricle as a response to the increased pressure. Ref: (28)

87. Diagnosis/ENT/Ophthalmology A 56 year-old female presents complaining of intense left eye pain associated with unilateral headache, nausea, and colored rings around lights. On examination you note decreased visual acuity, a pupil that is fixed and mid-dilated, and ciliary flushing. Which of the following is the most likely diagnosis? A. Acute glaucoma B. Migraine C. Episcleritis D. Acute uveitis Explanations (c) A. Acute glaucoma is an ocular emergency that presents as an acutely painful eye and elevated intraocular pressure. Patients typically complain of acute eye pain associated with unilateral headache, nausea/vomiting, cloudy vision, and colored rings around lights. On exam the pupil is fixed and mid dilated with prominent ciliary flush. (u) B. Migraine headaches have associated unilateral headache and nausea however there would be no pupillary changes. (u) C. Episcleritis is an inflammation of the thin layer of connective tissue between the conjunctiva and sclera. Episcleritis resembles conjunctivitis but is a more localized process and discharge is absent. (u) D. Acute uveitis is frequently due to systemic disorders associated HLA-B27-related conditions ankylosing spondylitis, reactive arthritis, psoriasis, ulcerative colitis, and Crohn's disease. The pupil is usually small, inflammatory cells and flare within the aqueous are present. Ref: (28) 88. Clinical Therapeutics/Pulmonology Topic: 1e Author: Donna Yeisley A 2 month-old infant has been diagnosed with pneumonia due to Chlamydia trachomatis. Which of the following is the treatment of choice? A. Ceftriaxone (Rocephin) B. Doxycycline C. Levofloxacin (Levaquin) D. Erythromycin Explanations (u) A. Ceftriaxone is a third-generation cephalosporin that may be safely used in children, however is not indicated for the treatment of Chlamydial pneumonia. (h) B. Doxycycline is a tetracycline and is contraindicated in children under eight years of age secondary to damaging effects on bone and teeth enamel. (h) C. Levofloxacin is a fluoroquinolone and is contraindicated for use in children under 18 years of age secondary to damaging effects that may occur with growing cartilage. (c) D. Erythromycin or sulfisoxazole is the treatment of choice for an infant with Chlamydial pneumonia. Ref: (13)

37

89. Clinical Intervention/Psychiatry/Behavioral Medicine Which of the following is the most appropriate intervention for a patient suffering from a specific phobia, such as fear of snakes? A. Lithium B. Behavioral therapy C. Insight-oriented therapy D. Electroconvulsive therapy Explanations (u) A. Lithium is indicated for treatment of bipolar disease, not phobias. (c) B. Specific phobias most commonly are treated with behavioral therapy including exposure therapy utilizing systemic desensitization. Hypnosis, supportive therapy and family therapy may also be useful adjunct treatment. (u) C. Insight-oriented therapy, while helpful in allowing patients to understand the origin of their fear and potential reasons for secondary gain, will not help to alleviate the patient's phobic symptoms. (u) D. The primary indication for electroconvulsive therapy (ECT) is in treatment of a major depressive disorder that has not responded to medical management. Ref: (14) 90. Health Maintenance/Endocrinology A patient is diagnosed with hypothyroidism and started on levothyroxine (Synthroid). When is it ideal to recheck the TSH level? A. Two weeks B. Three to four weeks C. Two to three months D. Six months Explanations (u) A. See B for explanation. (c) B. Patients taking levothyroxine (Synthroid) for thyroid replacement will achieve peak levels of T4 within three to four weeks. The half-life of levothyroxine is 7 days so it will take three to four weeks in order to achieve a steady state which means that TSH levels or T4 levels should not be checked sooner than this recommended time of three to four weeks. (u) C. See B for explanation. (u) D. See B for explanation. Ref: (28) 91. Diagnostic Studies/Gastrointestinal/Nutritional A patient presents uncertain as to whether he has completed the Hepatitis B vaccination series. What laboratory test would be most helpful in determining his immunization status?

A. Anti-HBs B. HBeAg C. HBsAg D. Anti-HBc Explanations (c) A. Anti-HBs is the circulating antibody that develops to surface antigen in response to either past Hepatitis B infection or to Hepatitis B immunization. It represents immunity to the Hepatitis B virus in otherwise immunocompetent patients. (u) B. HBeAg is seen with active disease and not from immunization. (u) C. The surface antigen to Hepatitis B (HBsAg) is most often used to show acute infection and not immunity. (u) D. Anti-HBc is seen with active disease and not from immunization. Ref: (7)

38

92. Diagnosis/Cardiology A 65 year-old female who recently had an anterior MI returns to clinic for follow-up six weeks after. She has no chest pain, but reports decreased exercise tolerance. Electrocardiogram (ECG) shows persistent ST elevation in leads V2-V4. Which of the following is the most likely diagnosis? A. Right ventricular infarction B. Re-occlusion of the right coronary artery C. Pericarditis D. Ventricular aneurysm Explanations (u) A. RV infarction is present in one-third of patients who had an inferior wall MI and typically show ST elevation in V4 along with the inferior leads. (u) B. Occlusion of the right coronary artery would show ST elevation in the inferior ECG leads. (u) C. Pericarditis shows ST elevation in multiple leads with a history consistent of a viral illness or fever. (c) D. Persistent ST elevation in the leads where a previous or recent anterior MI occurred is most likely due to ventricular aneurysm. Ref: (28) 93. History & Physical/Pulmonology Which of the following is a physical examination finding that is consistent with a diagnosis of lobar pneumonia? A. Resonant to percussion B. Late inspiratory crackles C. Decreased tactile fremitus D. Tracheal shift toward uninvolved side Explanations (u) A. Physical examination findings that are consistent with a diagnosis of lobar pneumonia include dullness to percussion, increased tactile fremitus and a trachea that is midline. (c) B. Late inspiratory crackles are a physical examination finding that is consistent with lobar pneumonia. (u) C. See A for explanation. (u) D. See A for explanation. Ref: (3) 94. Clinical Therapeutics/Urology/Renal A 35 year-old male presents with fever, perineal pain, and dysuria. On physical examination, the patient is toxic-appearing, febrile, and his prostate is very tender to palpation. Laboratory testing reveals leukocytosis, pyuria, and bacteriuria. Which of the following is the treatment of choice for this patient? A. Ampicillin and gentamicin B. Ceftriaxone and doxycline C. Trimethoprim-sulfamethoxazole D. Nitrofurantoin Explanations (c) A. Acute prostatitis is best treated acutely with parenteral antibiotics, such as ampicillin and gentamicin. (u) B. Ceftriaxone and doxycycline are used in the treatment of acute epididymitis due to sexually transmitted infection. (u) C. Trimethoprim-sulfamethoxazole can be used but is second line in toxic patients and is best used after the patient is stable. (u) D. Nitrofurantoin is used in the treatment of acute cystitis and not indicated in acute prostatitis. Ref: (28)

39

95. Clinical Intervention/Orthopedics/Rheumatology A 57 year-old male presents with acute bilateral lower extremity weakness and urinary incontinence that began after he fell earlier today. His examination is significant for bilateral lower extremity sensory deficits and weakness along with decreased rectal sphincter tone. Which of the following is the most appropriate intervention? A. Epidural steroids B. Oral NSAIDs C. Physical therapy D. Surgery Explanations (h) A. While epidural steroids can be effective in treating lumbar disc herniation, in the case of cauda equina syndrome, immediate surgical decompression is mandatory. (h) B. NSAIDs may be beneficial in some cases of lumbar muscle strain and disc herniation. They are not appropriate for management of cauda equina syndrome, immediate surgical decompression is mandatory. (h) C. Physical therapy may be beneficial in some cases of lumbar muscle strain and disc herniation but it is not appropriate for management of cauda equina syndrome. (c) D. Cauda equina syndrome is a rare but serious surgical emergency because the duration of nerve compression is inversely correlated with the likelihood of full neurologic recovery. Ref: (27) 96. Health Maintenance/Neurology A 45 year-old male presents for a routine physical. His mother and father both had ischemic strokes in their 70's. He does not smoke. His blood pressure is 128/80 mmHg, pulse 78/minutes and regular, respiratory rate of 12/minute. He has no bruits on examination. Which of the following studies should be ordered to further evaluate this patient's risk of stroke? A. Electrocardiogram B. Fasting lipid profile C. Carotid Doppler ultrasound D. MRI with gadolinium Explanations (u) A. ECG does not show risk of stroke. (c) B. This patient has a genetic risk for stroke secondary to family history. Having an elevated LDL cholesterol and low HDL cholesterol puts him at a greater risk for stroke. (u) C. Carotid Doppler US will demonstrate the presence of a plaque in the carotid arteries, but still does not evaluate stroke risk. (u) D. MRI with contrast will only show abnormalities in structure which cannot predict risk of stroke. Often abnormal findings are picked up on MRI that have no relationship to a patient's symptoms. Ref: (7) 97. Scientific Concepts/Obstetrics/Gynecology Mastitis associated with breastfeeding is most commonly caused by what bacteria? A. Listeria monocytogenes B. Escherichia coli C. Staphylococcus aureus D. Streptococcus pyogenes Explanations (u) A. See C for explanation. (u) B. See C for explanation. (c) C. The most common pathogen associated with postpartum mastitis is Staphylococcus aureus which arises from the nursing infant's throat and nose. (u) D. See C for explanation. Ref: (4)

40

98. Diagnosis/Gastrointestinal/Nutritional A patient with a history of severe peptic ulcer disease is 5 weeks status post Billroth I surgery. One week ago he restarted his normal diet and has had the onset of severe nausea, abdominal cramping, and light-headedness that occur approximately thirty minutes after eating. The abdominal exam reveals a healing surgical scar without areas of unusual tenderness or any palpable masses. Which of the following is the most likely diagnosis? A. Anxiety disorder B. Celiac sprue C. Dumping syndrome D. Irritable bowel syndrome Explanations (u) A. Anxiety disorders can cause a wide variety of somatic syndromes such as those mentioned, but generally not with such a straightforward dietary trigger. (u) B. Celiac sprue can cause similar symptoms as those listed, can develop at any age and can be worsened by the ingestion of gluten containing products. The patient's recent surgery makes dumping syndrome a much greater possibility. (c) C. Dumping syndrome typically occurs after Billroth type I surgeries as well as gastric bypass surgeries when the patient attempts to eat a large amount of simple sugars. (u) D. Irritable bowel syndrome is a diagnosis of exclusion and is associated with alternation in bowel habits. Ref: (7) 99. Clinical Therapeutics/ENT/Ophthalmology An 18 year-old patient is diagnosed with bacterial conjunctivitis. Gram stain reveals gram-negative intracellular diplococci. Which of the following is the most appropriate treatment of this infection? A. Gentamicin B. Penicillin C. Bactrim D. Ceftriaxone Explanations (u) A. Gentamicin is used in the treatment of gram negative rods and does not provide coverage of Neisseria gonorrhoeae. (u) B. Penicillin does provide coverage for some Neisseria species but is not considered the treatment of choice for Neisseria gonorrhoeae because of increased resistance. (u) C. Bactrim is used to treat Chlamydia trachoma not Neisseria gonorrhoeae. (c) D. Neisseria gonorrhoeae is confirmed by the Gram stain findings of intracellular gram-negative diplococci. The treatment of choice for Neisseria gonorrhoeae is ceftriaxone. Ref: (28) 100. History & Physical/Orthopedics/Rheumatology Which of the following is the most sensitive to determine whether there is a small effusion in the knee? A. Compress the patella and move it against the femur, noting any crepitus B. Flex the knee to about 90 degrees and palpate for tenderness over the joint line C. Milk the medial aspect of the knee, press lateral margin of the patella, and note a bulge of returning fluid medial to the patella D. Force fluid into space between the patella and the femur, tap the patella over the femur to detect a click Explanations (u) A. Crepitus without pain is not significant and does not indicate an effusion. (u) B. Tenderness over the joint line indicates a meniscal injury, but does not demonstrate an effusion. (c) C. A small bulge of returning fluid after milking fluid upward from the knee is useful for detecting small effusions. (u) D. Ballottement of the patella against the femur is useful for detecting large effusions, but not small ones. Ref: (3)

41

101. Diagnostic Studies/Cardiology A 15 year-old male is brought to the emergency department 1 hour after an episode of syncope while running in a 400-meter race. He had a similar episode 2 years ago. His mother and maternal first cousin died suddenly at the ages of 32 and 17 years, respectively. Examination shows abrasions of the face, hands, and knees. Neurologic examination shows no abnormalities. Which of the following is the most appropriate initial test? A. Tilt table test B. CT scan of the head C. Electrocardiogram D. Electroencephalography Explanations (u) A. Tilt table testing should be performed after structural heart disease has been ruled out. (u) B. CT scanning of the head would not be the initial test of choice. (c) C. ECG is the most appropriate to rule out any underlying rhythm abnormalities, Holter or event monitoring would also be included. (u) D. EEG testing routinely would not be helpful as an initial test in this patient. Ref: (11) 102. Clinical Intervention/Pulmonology A 42 year-old male is brought to the emergency department with a stab wound to his right lateral chest wall. On physical examination, the patient is stable with decreased breath sounds on the right with dullness to percussion. An upright chest x-ray reveals the presence of a moderate pleural effusion. Subsequent diagnostic thoracentesis contains bloody aspirate. Which of the following is the next most appropriate intervention? A. Thoracotomy B. Needle aspiration C. Close observation D. Tube thoracostomy Explanations (u) A. A thoracotomy is indicated in a small percentage of patients that do not respond to IV administration of fluids and evacuation of the hemothorax via tube thoracostomy. (u) B. Needle aspiration as treatment for a hemothorax is not recommended as it fails to adequately drain the bloody fluid. (u) C. Close observation is only indicated for small effusions in a patient that remains hemodynamically stable. (c) D. This patient has a hemothorax. Drainage of a hemothorax is best obtained through insertion of a chest tube (tube thoracostomy). Ref: (29) 103. Health Maintenance/Infectious Diseases Routine prophylaxis for patients going to an area of malaria infestation requires the administration of which chemoprophylaxis agent? A. Chloroquine B. Ciprofoxacin C. Tetracycline D. Erythromycin Explanations (c) A. The use of Chloroquine is still recommended as the drug of choice for prophylaxis in areas of non-resistant falciparum malaria. (u) B. The use of quinolones, tetracyclines, and macrolides is not recommended for the prevention of malaria. (u) C. See B for explanation. (u) D. See B for explanation. Ref: (28)

42

104. Diagnosis/Orthopedics/Rheumatology A 41 year-old female complains of 3 weeks of gradually worsening pain at the base of the thumb and radial aspect of the wrist. She and her husband have been renovating their home for the past 2 months and it has become increasingly difficult for her to hold a hammer. She denies numbness or tingling. She denies any history of previous trauma to the wrist. On examination, there is tenderness over the distal radial styloid and pain reproduced with ulnar deviation of a fist clenched over the abducted thumb. Which of the following is the most likely diagnosis? A. Carpal tunnel syndrome B. deQuervain's tenosynovitis C. Ganglion cyst D. Volar flexor tenosynovitis Explanations (u) A. Carpal tunnel syndrome typically presents with pain and paresthesias in the median nerve distribution. (c) B. deQuervain's tenosynovitis typically results from repetitive activity involving pinching the thumb while moving the wrist. There is often pain and tenderness over the radial styloid and Finkelstein's is positive in this patient. (u) C. Ganglion cysts classically present with a visible or palpable, usually painless swelling over the dorsum of the wrist. (u) D. With volar flexor tenosynovitis, pain is expected with extension of the fingers and localized tenderness of the volar tendon sheaths. Ref: (19) 105. Scientific Concepts/Urology/Renal Which of the following is the most common cause of acute epididymitis in men under the age of 40? A. Chlamydia trachomatis B. Ureaplasma urealyticum C. Pseudomonas aeruginosa D. Escherichia coli Explanations (c) A. In men under the age of 40, acute epididymitis is typically caused by Chlamydia trachomatis and Neisseria gonorrhoeae. (u) B. See A for explanation. (u) C. See A for explanation. (u) D. See A for explanation. Ref: (28) 106. History & Physical/Dermatology The bite from which of the following is associated with fever, lacrimation, rhinorrhea, bradycardia, hypertension, and tachyarrhythmias? A. Brown recluse spider B. Black widow spider C. Black flies D. Bedbugs Explanations (u) A. Brown recluse spider bites may cause reactions ranging from mild urticaria to full thickness necrosis but typically, there are no systemic symptoms. (c) B. Black widow spider bites can inject venom that contains a neurotoxin which can produce reactions at the site of the bite along with varying degrees of systemic symptoms. (u) C. Black fly bites produce local reactions as well as fever, nausea, and general lymphadenopathy. (u) D. Bedbugs have nocturnal feedings that produce a linear arrangement of papular urticaria. There are no systemic symptoms associated with their bites. Ref: (10)

43

107. Clinical Therapeutics/Cardiology What is the optimal INR for a patient with a mechanical mitral valve prosthesis on warfarin (Coumadin)? A. 1.5-2.5 B. 2.0-3.0 C. 2.5-3.5 D. 3.5-4.5 Explanations (u) A. See C for explanation. (u) B. See C for explanation. (c) C. Patients with mechanical MV prostheses should maintain an INR between 2.5-3.5. Aortic mechanical valves can be maintained at an INR of 2.0-3.0. (u) D. See C for explanation. Ref: (28) 108. Clinical Intervention/Obstetrics/Gynecology A 40 year-old female presents with a Pap smear abnormality revealing atypical glandular cells (AGUS). What is the most appropriate intervention? A. HPV DNA testing B. Colposcopy with endometrial curretage(ECC) C. Repeat Pap smear in 3 months D. Colposcopy and endometrial sampling Explanations (u) A. HPV DNA testing is recommended to further evaluate patients with Pap smears with dysplasia. (u) B. Colposcopy with ECC would be recommended in patients with ASCUS, LGSIL, HGSIL, or squamous cell findings on a Pap smear. (u) C. Repeat Pap smear would be recommended in patients with ASCUS, not AGUS, results on a Pap smear. (c) D. Colposcopy and endometrial sampling are important to perform in patients with AGUS Pap results because glandular cells are associated with squamous and glandular precursor lesions and carcinoma. Ref: (4) 109. Diagnostic Studies/ENT/Ophthalmology Which of the following diagnostic studies would be most helpful in diagnosing a retropharyngeal abscess? A. CBC B. Neck CT scan C. Rapid strep screen D. Heterophile antibody Explanations (u) A. A CBC would be an important test, but it is not definitive for peritonsillar abscess. (c) B. A neck CT scan would identify a peritonsillar abscess. (u) C. A rapid strep screen may have been performed prior to placement on antibiotics, but does not provide a definitive diagnosis for a peritonsillar abscess. (u) D. A heterophile antibody is not indicated in the diagnosis of a retropharyngeal abscess. Ref: (7) 110. Health Maintenance/Psychiatry/Behavioral Medicine Which of the following is the leading cause of injury-related death in children between the ages of 1 and 15? A. Drowning B. Firearms C. House fires D. Motor vehicle injuries

44

Explanations (u) A. Drowning is a frequent, but not the leading, cause of death in early childhood. (u) B. Childhood death due to firearm related injuries is becoming more common, but it is not the leading cause. (u) C. House fires are a common, but not the leading, cause of death in early childhood. (c) D. Motor vehicle injuries are the leading cause of death in children. Ref: (13) 111. Diagnosis/Pulmonology A 45 year-old male presents with complaints of a chronic cough productive of mucopurulent sputum. The cough has been present for the past 3 years, but he attributed it to a "smoker's cough". He has been coughing up a lot of sputum lasting all winter long for the past 2 years. He denies any hemoptysis, weight loss or chest pain. Physical examination reveals a moderately obese male in no acute respiratory distress. Lung fields reveal presence of scattered rhonchi and wheezes. There is 1+ peripheral edema. Which of the following is the most likely diagnosis? A. Lung cancer B. Bronchiectasis C. Chronic bronchitis D. Interstitial lung disease Explanations (u) A. While the respiratory complaints of lung cancer are associated with the location and type of primary tumor, anorexia and weight loss is seen in the majority of patients. Patients will also usually have a new cough or a change in a chronic cough and may complain of hemoptysis and nonspecific chest pain. (u) B. While bronchiectasis presents with a chronic cough productive of copious amounts of purulent sputum, these patients most commonly also have associated complaints of hemoptysis, weight loss and pleuritic chest pain. Examination of the lungs reveals persistent crackles at the bases. (c) C. This patient most likely has chronic bronchitis which is defined as sputum production and cough for at least 3 months of the year for 2 consecutive years which is primarily caused by cigarette smoking. (u) D. Interstitial lung disease is characterized by progressive exertional dyspnea and cough, however sputum production is minimal and the examination of the lungs reveals fine, late inspiratory crackles at the bases in the majority of patients. Ref: (28) 112. Clinical Therapeutics/Neurology A 72 year-old man with a long-standing history of diabetes mellitus, renal insufficiency, and hypertension presents to the clinic complaining of burning and tingling pain in his feet. Which of the following medications would help control pain in this patient? A. Phenobarbital B. Amitriptyline (Elavil) C. Celecoxib (Celebrex) D. Codeine Explanations (u) A. This is an antiepileptic medication that does not help diabetic neuropathy. (c) B. Amitriptyline has been recommended for pain associated with diabetic neuropathy. (h) C. Celecoxib is not indicated for diabetic neuropathy and should not be used if there is decreased kidney function. (u) D. Treatment with codeine puts the patient at risk for dependence and this does not help the actual neuropathic pain. Ref: (28) 113. History & Physical/Gastrointestinal/Nutritional A patient presents with abdominal pain in the right lower quadrant, examination reveals increased pain in the right lower quadrant on deep palpation of the left lower quadrant. This commonly known as which of the following? A. Psoas sign B. Murphy's sign C. Rovsing's sign D. Obturator sign

45

Explanations (u) A. Psoas sign is right lower quadrant pain with right leg extension. (u) B. Murphy's sign is seen in liver and gallbladder disease in which the patient abruptly halts deep inspiration due to discomfort as the examiners hand applies pressure to the right upper quadrant. (c) C. A positive Rovsing's sign can be elicited in a patient with appendicitis when increased pain occurs in the right lower quadrant upon palpation of the left lower quadrant. (u) D. Obturator sign is right lower quadrant pain with internal rotation of the hip. Ref: (3) 114. Clinical Intervention/Cardiology Which of the following can optimize quality of life and is an definitive treatment for a patient with refractory heart failure? A. Ventricular assist device B. Intra-aortic balloon counterpulsation C. Cardiac transplantation D. Partial resection of the left ventricle Explanations (u) A. Ventricular assist devices can help to provide temporary circulatory support to those awaiting transplantation. (u) B. IABC is useful in cardiogenic shock, but does not improve quality of life. (c) C. Cardiac transplantation is effective, with survival rates of 80-90% in 1 year, 60-70% over 5 years. It does improve quality of life despite the immunosuppression medications. (u) D. Partial resection of the left ventricle is still experimental and has not shown to have successful results. Ref: (11) 115. Scientific Concepts/Orthopedics/Rheumatology A football player complains of burning pain, numbness, and tingling extending from the left shoulder down into the hand after he tackled a player. These symptoms resolved spontaneously in minutes. Following resolution of the symptoms, he has full strength and normal sensation in the left arm. What is the most likely etiology of his symptoms? A. Acute muscle strain B. Mild concussion C. Stretching of nerve roots and brachial plexus D. Thoracic outlet obstruction Explanations (u) A. Acute cervical muscle strain might result from the mechanism of injury described but symptoms would not quickly resolve. (u) B. Concussion involves generalized symptoms such as loss of consciousness but would not affect one limb exclusively. (c) C. Brachial plexus neurapraxia, commonly called "stinger" injuries, results from stretching of the cervical nerve roots and brachial plexus by a mechanism such as that described in this question. (u) D. Thoracic outlet syndrome is most commonly caused by cervical rib and is usually only symptomatic when the arm is elevated. Ref: (26) 116. Diagnostic Studies/Hematology A positive osmotic fragility test is seen in which of the following? A. G6PD deficiency B. Sickle cell anemia C. Hereditiary spherocytosis D. Autoimmune hemolytic anemia

46

Explanations (u) A. G6PD deficiency may result in episodic hemolysis in response to oxidant drugs or infections and is not associated with a positive osmotic fragility test. (u) B. Sickle cell anemia is an abnormal hemoglobinopathy leading to chronic hemolytic anemia and is confirmed with hemoglobin electrophoresis. An osmotic fragility test would be negative. (c) C. Hereditary spherocytosis will have a positive osmotic fragility test secondary to a defective red blood cell membrane. (u) D. Coombs' testing forms the basis for the diagnosis of autoimmune hemolytic disorders. Ref: (28) 117. Diagnostic Studies/Endocrinology Patients with primary adrenal insufficiency will have which of the following electrolyte abnormalities? A. Hyperkalemia B. Hypernatremia C. Hypercalcemia D. Hyperosmolality Explanations (c) A. Patients with primary adrenal insufficiency will not have the adrenal gland producing cortisol, aldosterone, and the sex hormones. As a result of the lack of the mineralocorticoid aldosterone, the kidney will not save sodium and will instead save potassium. (u) B. Patients with primary adrenal failure will lack production of aldosterone and will not be able to save sodium. (u) C. Patients with hyperparathyroidism will have an increase in the serum calcium level as a result of the parathyroid gland retaining calcium. (u) D. Increased serum osmolality is the result of diabetes insipidus which results in a dilute urine being produced with resultant increase in the serum osmolality, as compared to the urine osmolality. Ref: (28) 118. Health Maintenance/Obstetrics/Gynecology What is the most common cause of secondary amenorrhea? A. Pregnancy B. Ovarian failure C. Imperforate hymen D. Hypothalamic amenorrhea Explanations (c) A. Pregnancy is the most common cause of amenorrhea and is essential to exclude by a serum or urine pregnancy test. (u) B. With ovarian failure, the ovarian follicles are resistant to stimulation. Ovarian failure can be caused by chromosomal abnormalities, premature menopause, or a complication of chemotherapy. (u) C. An imperforate hymen would prevent menstrual bleeding, this is a cause of primary amenorrhea. (u) D. The pulsatile release of GnRH is disrupted and the anterior pituitary gland is not stimulated to release FSH and LH. This can be caused by different etiologies including: weight loss, weight gain, excessive exercise, drug induced, tumors, anorexia, and other chronic medical illnesses. Ref: (4)

119. History & Physical/Cardiology Which of the following physical exam findings suggests worsening or severe aortic stenosis? A. An ejection click preceding the murmur B. A split S2 with respiration variation C. Grade 2/6 systolic murmur radiating to the carotids D. Palpable thrill over the right second intercostal space

47

Explanations (u) A. This finding is typical in mild to moderate AS. (u) B. This can be a normal finding. (u) C. This is the typical murmur for aortic stenosis, but does not suggest the degree of severity by the grade. (c) D. A palpable thrill or LV heave with associated murmur suggests severe AS. Ref: (28) 120. Clinical Therapeutics/Urology/Renal A 60 year-old male presents with difficulty voiding and having to get up twice a night to urinate. On physical examination, the prostate is firm, smooth, and enlarged. Prostate specific antigen level is normal. Which of the following is the best treatment option for this patient? A. Leuprolide (Lupron) B. Terazosin (Hytrin) C. Ciprofloxacin (Cipro) D. Naproxen (Naprosyn) Explanations (u) A. Leuprolide is a luteinizing hormone-releasing hormone agonists used in the treatment of prostate cancer. (c) B. Terazosin is an alpha-blocker and used to treat benign prostatic hyperplasia. (u) C. Ciprofloxacin is an antibiotic used to treat acute prostatitis not benign prostatic hypertrophy. (u) D. Naproxen is an anti-inflammatory agent and not used in the treatment of benign prostatic hypertrophy. Ref: (28) 121. Clinical Intervention/ENT/Ophthalmology In patents with diabetic retinopathy, what clinical intervention is most successful in preserving vision? A. Panretinal laser photocoagulation B. Iridectomy C. Radial keratotomy D. Vitrectomy Explanations (c) A. Panretinal laser photocoagulation is indicated for preservation of vision in patients with diabetic retinopathy. (h) B. Iridectomy is of no value in preserving the retina and iridectomy is harmful in this situation due to the trauma it causes to the eye. (h) C. Radial keratotomy is indicated to correct myopia. This surgery destroys normal eye architecture and has no benefit in diabetic retinopathy. (h) D. Vitrectomy is indicated for treatment of retinal tears and not to preserve an intact retina. Ref: (7) 122. Diagnostic Studies/Neurology A 53 year-old female with sudden onset "of the worst headache she has ever had" presents to the emergency department. She has a history of migraines but states that the current headache is not like her usual headaches. Results of her physical examination are unremarkable. Which of the following is the next best step in the evaluation of this patient? A. Angiogram B. CT scan C. Transcranial Doppler D. MRI Explanations (u) A. Angiogram is necessary to define details of aneurysm and anatomic configuration, but not as an initial diagnostic study. (c) B. This patient's history is highly suggestive of subarachnoid hemorrhage. CT is best to screen for intracranial hemorrhage. It is faster than MRI and more sensitive in the first 24 hours. (u) C. Transcranial Doppler can detect cerebral artery vasospasm but cannot detect aneurysm. (u) D. MRI is not as sensitive for an acute bleed, but is appropriate for old bleeds. Ref: (28)

48

123. Diagnosis/Gastrointestinal/Nutritional A 45 year-old male presents with a long history of ulcerative colitis and recent progressive complaints of right upper quadrant pain, weight loss, fever and most recently, a rapid onset of jaundice with pruritus. Labs revealed elevated bilirubin and alkaline phosphatase. Viral serologies were negative. An endoscopic cholangiogram showed areas of stenosis and dilation throughout the bile duct system. What is the most likely diagnosis? A. Choledocholithiasis B. Hepatic carcinoma C. Portal hypertension D. Primary sclerosing cholangitis Explanations (u) A. Choledocholithiasis can most certainly cause elevated bilirubin and other liver function tests when obstruction occurs. It also occurs more often in persons with sclerosed or narrowed bile ducts however it does not generally carry a poor prognosis and can be generally recognized by its typical symptoms. (u) B. Hepatic carcinoma does not cause areas of varied stenosis and dilation in the biliary tree. (u) C. Portal hypertension may present with jaundice but generally has significant ascites as well that helps to distinguish this disorder. It also does not result in the bile duct pattern mentioned. (c) D. Primary sclerosing cholangitis (PSC) results in diffuse intra- and extrahepatic duct sclerosing with dilatation proximal to these areas. Ref: (7) 124. Clinical Therapeutics/Orthopedics/Rheumatology Topic: 6a Author: Jonathan Bowser A 43 year-old female presents with a two year history of frequent episodes of pain and morning stiffness in both hands and wrists. She experiences some symptomatic relief with ibuprofen, but feels that the episodes are becoming more frequent and severe. On examination, you observe joint swelling of several MCP joints on both hands. X-ray of the hands shows joint space narrowing of the MCP joints. In addition to NSAIDs, what is the most appropriate first-line long-term medication to treat this patient? A. Prednisone (Deltasone) B. Infliximab (Enbrel) C. Methotrexate (Rheumatrex) D. Sulfasalazine (Azulfidine) Explanations (u) A. Prednisone can produce impressive clinical improvement in rheumatoid arthritis but the side effects associated with their long-term use limit their utility. (u) B. Tumor necrosis factor inhibitors, such as infliximab, are very effective disease modifying antirheumatic drugs (DMARDS). This class of medication is inappropriate as a first-line medication due to very high cost. (c) C. Methotrexate is the standard first-line medication in the treatment of rheumatoid arthritis. This DMARD is generally effective, well-tolerated, and affordable. (u) D. Sulfasalazine is inexpensive but associated with potentially serious hematologic side effects and is considered a second-line DMARD. Ref: (28) 125. Scientific Concepts/Cardiology Perfusion of the coronary arteries occurs primarily during which of the following? A. Systole B. Diastole C. Afterload D. Preload Explanations (u) A. See B for explanation. (c) B. Coronary artery perfusion occurs primarily during diastole. (u) C. See B for explanation. (u) D. See B for explanation. Ref: (11)

49

126. History & Physical/Pulmonology Which of the following is a common presenting clinical manifestation of a patient with interstitial lung disease? A. Early inspiratory crackles B. Progressive dyspnea on exertion C. Productive cough with copious sputum D. Decreased breath sounds with hyperresonant percussion Explanations (u) A. Late, not early, inspiratory crackles are associated with interstitial lung disease. (c) B. Patients with interstitial lung disease commonly present with progressive dyspnea on exertion and a cough with minimal sputum production. (u) C. A productive cough of copious amounts of sputum is most typical of a patient with chronic bronchitis. (u) D. Physical examination findings of decreased breath sounds with hyperresonant percussion is consistent with a diagnosis of chronic obstructive lung disease. Ref: (28) 127. Clinical Intervention/Psychiatry/Behavioral Medicine A patient is started on a new antipsychotic medication for his disorder. Three days later he develops altered consciousness, lead-pipe rigidity, diaphoresis and catatonia. Vital signs reveal respiratory rate of 20, temperature of 105.6 degrees F, and pulse oximetry of 95% room air. Which of the following would be the most appropriate initial intervention in this patient? A. Immediate oral SSRI's and Benadryl B. Supportive care with fluids and antipyretics C. IV antibiotics and naloxone D. Intubation and mechanical ventilation Explanations (u) A. See B for explanation. (c) B. Neuroleptic malignant syndrome is characterized by extrapyramidal signs, blood pressure changes, altered consciousness, hyperpyrexia, muscle rigidity, dysarthria, cardiovascular instability, fever, pulmonary congestion and diaphoresis. Controlling fever and fluid support are the best initial management. With a normal pulse oximetry mechanical ventilation is not indicated. (u) C. See B for explanation. (u) D. See B for explanation. Ref: (28) 128. Diagnosis/ENT/Ophthalmology A patient presents with a nontender, painless, nodule involving a meibomian gland. Which of the following is the most likely diagnosis? A. Chalazion B. Dacryocystitis C. Entropion D. Hordeolum Explanations (c) A. Chalazion is characterized by a hard, nontender swelling on the upper or lower lid with redness and swelling of the adjacent conjunctiva and is due to granulomatous inflammation of a meibomian gland. (u) B. Dacryocystitis is an infection of the lacrimal sac due to obstruction of the nasolacrimal system. (u) C. Entropion is an outward turning of the lower lid. (u) D. A hordeolum is a bacterial inflammation of the base of the eyelash. Ref: (3)

50

129. Health Maintenance/Gastrointestinal/Nutritional Which of the following is the most effective prophylaxis for traveler's diarrhea? A. Metronidazole B. Ciprofloxacin C. Tetracycline D. Ampicillin Explanations (u) A. Ampicillin, tetracycline, and metronidazole do not cover the common causes of traveler's diarrhea. (c) B. Options for prophylaxis of traveler's diarrhea include norfloxacin, ciprofloxacin, ofloxacin, or trimethoprim/sulfamethoxazole. (u) C. See A for explanation. (u) D. See A for explanation. Ref: (28) 130. Diagnostic Studies/Obstetrics/Gynecology Which of the following tests is the most specific for the diagnosis of syphilis? A. Rapid plasma reagin (RPR) B. Weil-Felix agglutination test C. Venereal Disease Research Laboratory (VDRL) D. Fluorescent treponemal antibody absorption (FTA-ABS) Explanations (u) A. The RPR is a non-specific test. False positives are common. (u) B. The Weil-Felix agglutination test is used for rickettsial infections not syphilis. (u) C. The VDRL if positive must by confirmed with an additional testing because of a large number of false positives including bacterial and viral infections, pregnancy, chronic liver disease, connective tissue disorders. (c) D. The FTA-ABS and the MTA-TP are specific treponemal tests used for the confirmation of syphilis. Ref: (4) 131. Scientific Concepts/Dermatology Which of the following types of infection is most likely to benefit from hyperbaric oxygen therapy? A. Clostridial infection B. Group A beta hemolytic streptococcus C. Serratia marcescens D. Methicillin resistant Staphylococcus aureus Explanations (c) A. Patients with clostridium myonecrosis (gas gangrene) will benefit from hyperbaric oxygen therapy, penicillin therapy, and radical surgical excision. (u) B. Patients with group A beta hemolytic strep, staph aureus, and community acquired methicillin resistant staph aureus will benefit from antibiotic therapy but these organisms are not anaerobic so hyperbaric oxygen therapy is not useful. (u) C. See B for explanation. (u) D. See B for explanation. Ref: (28)

132. History & Physical/Urology/Renal Which of the following is most frequently associated with renal cell carcinoma? A. Hematuria B. Inguinal pain C. Hypocalcemia D. Fever Explanations (c) A. Gross or microscopic hematuria, flank pain, or mass is common in renal cell carcinoma. (u) B. See A for explanation. (u) C. Hypercalcemia, not hypocalcemia, may be noted in patients with renal cell carcinoma. (u) D. Fever is uncommon in renal cell carcinoma and typically only noted with advanced disease. Ref: (28)

51

133. Clinical Therapeutics/Cardiology What is the mechanism of action for aspirin? A. Inhibits platelet aggregation by blocking cyclooxygenase-1 activity B. Exerts antiplatelet effects by blocking the platelet membrane adenosine diphosphate receptors C. Inhibits the platelet membrane glycoprotein IIb/IIIa receptor D. Converts plasminogen to the natural fibrinolytic agent plasmin Explanations (c) A. Aspirin inhibits platelet aggregation by blocking cyclooxygenase-1 activity. (u) B. This the mechanism of action for ADP antagonists such as clopidogrel or ticlopidine. (u) C. This is the mechanism of action for glycoprotein IIb/IIIa receptors such as abciximab (ReoPro). (u) D. This is the mechanism of action for tissue plasminogen activators (tPA). Ref: (11) 134. Clinical Intervention/Pulmonology A 68 year-old male with history of COPD is brought to the emergency department following a motor vehicle collision. On physical examination there is evidence of head trauma. The left side of the chest wall appears to move inward with inspiration and outward with expiration. A chest x-ray reveals multiple rib fractures on the left. Which of the following is the most appropriate intervention? A. Surgical fixation of the fractured ribs B. Application of elastic binders and adhesive tape C. Endotracheal intubation and mechanical ventilation D. Chest physiotherapy that encourages frequent coughing Explanations (u) A. Surgical fixation of flail chest is less reliable than positive-pressure ventilation and is performed only rarely in the United States. (u) B. While application of elastic binders and adhesive tape was historically utilized to stabile the chest, this intervention has been found to decrease chest expansion and worsen lung atelectasis. (c) C. Indications for early endotracheal intubation and mechanical ventilation in treatment of flail chest include patients that are over the age of 65, have comorbid lung disease and associated severe head trauma. Other indications include shock, three or more associated injuries and fracture of eight or more ribs. (u) D. Conservative treatment for mild to moderate flail chest includes analgesic relief of pain, chest physiotherapy that encourages frequent coughing and restriction of fluids to prevent fluid overload, however this patient needs early ventilatory support. Ref: (29) 135. Diagnosis/Orthopedics/Rheumatology A 13 year-old girl reports two weeks of worsening right knee pain with no history of antecendent injury or recent trauma. She reports frequent episodes of nighttime awakening with knee pain in the past two weeks. Examination of the knee reveals edema and a tender mass over the anterior proximal right tibia. Her knee exam is otherwise within normal limits. Radiographs of the right knee show a lytic mass with a multi-laminated periosteal reaction involving the proximal anterior tibia. What is the most likely diagnosis? A. Ewing sarcoma B. Osteochondroma C. Multiple myeloma D. Osteoid osteoma Explanations (c) A. The distinctive feature of Ewing sarcoma is the radiographic appearance of a periosteal "onion skin" reaction. (u) B. This benign tumor typically presents as a painless mass and appears in plain film radiographs as a stalk or broad-based projection from the surface of the bone. (u) C. The classic radiographic appearance of multiple myeloma is a lytic lesion but this is a condition that is seen in a much older population and is more likely to present with back pain. (u) D. Although the presentation may be similar to Ewing's sarcoma, the radiographs in osteoid osteoma typically show a round lucency surrounded by sclerotic bone. Ref: (5)

52

136. Diagnostic Studies/ENT/Ophthalmology In a patient with amaurosis fugax what is the most appropriate initial diagnostic study? A. Ophthalmoscopy B. Schiotz tonometry C. MR angiography D. Carotid ultrasound Explanations (u) A. Ophthalmoscopy provides visualization of retina but does not help elucidate the source of the plaque. (u) B. Schiotz tonometry is used to measure intraocular pressure. (u) C. MR angiography is a useful test to identify retinal vascular anatomy but is not the first choice in the search for causes of amaurosis fugax. (c) D. The most common cause of amaurosis fugax is an atherosclerotic plaque in the carotid artery which can be identified with ultrasound. Ref: (28) 137. Health Maintenance/Hematology An adult patient who has undergone a splenectomy secondary to idiopathic thrombocytopenic purpura should receive which prophylactic agent? A. Bicillin 1.2 million units IM every month B. Pneumococcal vaccine IM one time only C. Gamma globulin IM every month D. Hepatitis B vaccine IM series Explanations (u) A. Patients who are being treated for rheumatic fever may receive Bicillin 1.2 million units every month for Group A beta-streptococcal prophylaxis. (c) B. Pneumococcal vaccine should routinely be given to all patients who undergo splenectomy for prophylaxis to Streptococcal pneumoniae. (u) C. Gamma globulin is routinely given to patients for prophylaxis to hepatitis exposure. (u) D. Hepatitis B vaccine has no specific indication in patients after splenectomy. Ref: (28) 138. History & Physical/Cardiology A 56 year-old male with a 30 pack-year smoking history presents with substernal chest pain. The pain is described as a pressure that radiates to his jaw. The pain has lasted consistently for 30 minutes with variable relief. His current medications include atorvastatin (Lipitor) and glyburide (Micronase). Which of the following aggravating or relieving factors about the pain would support the diagnosis? A. Precipitated by exercise and relieved with rest B. Unrelieved by nitroglycerin C. Aggravated by deep breaths D. Relieved with food Explanations (u) A. Precipitated by exercise and relief with rest is consistent with angina. (c) B. Patient is having a myocardial infarction which is unrelieved by rest or nitroglycerin. (u) C. Aggravated by deep breaths suggests pericarditis. (u) D. Relieved with food suggests peptic ulcer disease. Ref: (11)

53

139. Clinical Therapeutics/Gastrointestinal/Nutritional A 30 year-old female presents with a one to two year history of daily bouts of mid-abdominal crampy pain and bloating that are briefly relieved with defecation. Loose to watery bowel movements occur four to five times daily. She denies any nocturnal symptoms. Previous laboratory studies, abdominal CT scans and a colonoscopy have all been normal. Attempted dietary changes, exercise and probiotics have provided no relief. What is the next step in treatment for this patient? A. Nortriptyline (Pamelor) B. Omeprazole (Prilosec) C. Polyethylene glycol (Miralax) D. Promethazine (Phenergan) Explanations (c) A. Nortriptyline, and other tricyclic antidepressants (TCAs), is an appropriate choice for someone with diarrhea predominant irritable bowel syndrome due to their numerous mechanisms of action. (u) B. Omeprazole, a proton pump inhibitor, works on parietal cells in the stomach lining to reduce production of hydrochloric acid. Though beneficial for peptic ulcer and reflux disease it would provide no relief of symptoms in a patient with IBS. (u) C. Polyethylene glycol is commonly used in constipation predominant IBS but would probably worsen this patient's symptoms. (u) D. The anticholinergic properties of the promethazine might be helpful even though its somnolent effects would be significant and would not be superior to nortriptyline. Promethazine's strength is in nausea control and less often its antihistamine affects. Ref: (28) 140. Diagnosis/Pulmonology A 28 year-old man presents to the emergency department complaining of sudden onset of shortness of breath associated with sharp right-sided chest pain increased with breathing. On physical examination, respirations are 20 per minute and blood pressure is 120/76 mm Hg. Auscultation of the chest reveals absent breath sounds over the right apex with normal heart sounds. Percussion of the right apex is noted to be hyperresonant. Which of the following is the most likely diagnosis? A. Hemothorax B. Pneumothorax C. Pulmonary embolus D. Foreign body aspiration Explanations (u) A. A hemothorax usually results from trauma. While breath sounds would be absent over the involved area, the percussion note would be dull, not hyperresonant. (c) B. This patient most likely has a spontaneous pneumothorax which is supported by the presenting symptoms of sudden onset of dyspnea and pleuritic chest pain as well as the physical exam findings of absent breath sounds and hyperresonance to percussion. (u) C. While a pulmonary embolism most commonly presents with dyspnea and pain on inspiration, the physical examination is often unremarkable and would not reveal the findings of absent breath sounds and hyperresonance to percussion. (u) D. Foreign body aspiration is most common in children and the elderly. Aspiration of a food bolus is the most common cause in adults which leads to a history of a choking episode followed by dysphagia, wheezing and coughing. Physical examination findings are dependent on the location of the obstruction. Ref: (28) 141. Clinical Intervention/Endocrinology In the treatment of alcohol-related hypoglycemia, thiamine is administered before glucose to prevent which of the following? A. Acute renal failure B. Wernicke's encephalopathy C. Korsakoff psychosis D. Mesenteric thrombosis

54

Explanations (u) A. Acute renal failure is prevented by adequate hydration allowing appropriate blood flow to the kidney. It is prevented by hydration with volume expanders rather than by thiamine and/or glucose. (c) B. Alcoholics are typically deficient in thiamine. If glucose alone is given to treat hypoglycemia, Wernicke's encephalopathy can be precipitated since thiamine is not available for nutritional purposes when glucose is replaced. Therefore, thiamine is always given prior to glucose until a satisfactory diet can be given in order to prevent Wernicke's encephalopathy. (u) C. Korsakoff psychosis is a sequelae of Wernicke's encephalopathy so if thiamine is used to prevent Wernicke's encephalopathy, its sequelae, Korsakoff's psychosis will also be prevented. (u) D. Mesenteric thrombosis is mostly a complication of atherosclerosis or embolism to the mesenteric artery. It is more common in low flow states such as hypotension and can somewhat be prevented by ensuring adequate hydration. It is not dependent on thiamine or glucose, rather it just needs volume expanders. Ref: (28) 142. History & Physical/Infectious Diseases You suspect that a 13 year-old patient has bacterial conjunctivitis. What physical examination finding would you expect? A. Crusting and matting of the eyelids with discharge and diffuse injection of the conjunctival surface B. Clear watery stringy discharge with conjunctival hyeremia and edema C. Circumcorneal injection of the conjunctiva D. Painless injection of the subconjunctival sclera bilaterally Explanations (c) A. Crusting and matting of the eyelids with discharge and diffuse injection of the conjunctival surface is a common presentation of a bacterial conjunctivitis. (u) B. Clear watery stringy discharge with conjunctival hyeremia and edema is a common presentation of an allergic conjunctivitis. (u) C. Circumcorneal injection of the conjunctiva is a common presentation of iridocyclitis. (u) D. Painless injection of the subconjunctival sclera bilaterally is a common presentation of episcleritis. Ref: (28) 143. Diagnostic Studies/Psychiatry/Behavioral Medicine A 52 year-old female complains of bouts of anxiety and depression, the latter very deep but short lived (<24 hours). She states "I often feel like I am going to jump out of my skin". Her periods have become less frequent over the past 6 months. She denies suicidal ideations. She is not sleeping through the night. Which of the following the most appropriate next step in this patient? A. Check a follicle-stimulating hormone level B. Start a tricyclic antidepressant C. Refer her to a psychiatrist D. Endometrial biopsy Explanations (c) A. An FSH level should be assessed in this patient to evaluate for menopause as a cause for her psychiatric symptoms. (u) B. See A for explanation. (u) C. See A for explanation. (u) D. Endometrial biopsy is indicated in a perimenopausal female with menorrhagia. Ref: (28) 144. Scientific Concepts/Obstetrics/Gynecology Maternal blood pressure normally decreases the most during what period of pregnancy? A. First trimester of pregnancy B. Second trimester of pregnancy C. Third trimester of pregnancy D. During labor and delivery

55

Explanations (u) A. Blood pressures during the first trimester will be similar to the non-pregnant state. (c) B. Diastolic blood pressure and the mean arterial pressure reach their nadir at 16-20 weeks of gestation. (u) C. Blood pressures during the third trimester will be similar to the non-pregnant state. (u) D. Blood pressure during labor and delivery often are variable. Blood pressure often rises with pain and apprehension. Blood pressure often decreases with the use of narcotics, epidurals, and significant blood loss. Ref: (4) 145. Diagnosis/Neurology A 32 year-old female presents with a seven month history of recurrent, brief episodes of weakness and tingling in the extremities, diplopia, and vertigo. Which of the following is the most likely diagnosis? A. Guillain-Barre syndrome B. Myasthenia gravis C. Multiple sclerosis D. Amyotrophic lateral sclerosis Explanations (u) A. Guillain-Barre syndrome typically presents with progressive weakness that starts peripherally and proceeds centrally over a brief period of time. (u) B. Myasthenia gravis presents with weakness and fatigue in the upper limbs, cranial, and/or trunk musculature. Blurry vision and diplopia are common visual complaints and dysphagia is common. (c) C. Multiple sclerosis is most frequently seen in patients in their twenties and presents with episodes of weakness, paresthesias, and diplopia. (u) D. Amyotrophic lateral sclerosis presents with progressive weakness, fasciculations, and loss of muscle mass. Ref: (28) 146. Clinical Therapeutics/Pulmonology An elderly nursing home resident is admitted with methicillin-resistant Staphylococcus aureus pneumonia. Which of the following is the most appropriate treatment to initiate? A. Nafcillin B. Vancomycin C. Clindamycin D. Doxycycline Explanations (u) A. Nafcillin is classified as a penicillinase-resistant penicillin, however would not be effective against methicillin-resistant strains of Staphylococcus aureus. (c) B. Vancomycin with or without the addition of gentamicin or rifampin and linezolid is the treatment of choice for methicillin-resistant Staphylococcus aureus. (u) C. Clindamycin is primarily utilized in treatment of severe anaerobic infections, but is not indicated for the treatment of methicillin-resistant Staphylococcus aureus. (u) D. Doxycycline is a tetracycline and is only utilized to treat respiratory infections that are susceptible to tetracycline, such as Mycoplasma or Chlamydial pneumonias. Ref: (28) 147. Health Maintenance/Cardiology A 26 year-old female presents to clinic with a left arm that is swollen and non-tender with bluish discoloration along the upper arm and forearm. She is status post pacemaker insertion in the left upper chest for third degree heart block, one week ago. Pulses are present and the arm is warm, but not red. The pacemaker incision is healing well despite a hematoma and tenderness at the site. Which of the following statements would be appropriate patient education about this condition? A. Reassurance that the discoloration is an expected finding. B. Apply cold compresses to the site of the hematoma. C. Elevation of the involved extremity will increase the swelling. D. Aspirin should be taken to help manage pain.

56

Explanations (c) A. This is indicative of migratory ecchymosis and expected after insertion of a pacemaker. (u) B. Warm compresses will help to decrease the hematoma. (u) C. Elevation will help to decrease the swelling. (u) D. ASA is not the pain medicine of choice in a patient with a hematoma due to its non-clotting properties. Ref: (11) 148. Clinical Intervention/ENT/Ophthalmology A 20 year-old presents 30 minutes after being struck by a hockey puck in the mouth. On physical examination a central incisor is missing from its socket. The patient has the tooth wrapped in tissue paper and the root appears intact. Which of the following is the most appropriate next step in the treatment of this patient? A. Administration of IM penicillin B. No treatment is warranted C. Place tooth in saline and refer to plastic surgery for reimplantation D. Immediately reimplant the tooth and refer to an oral surgeon Explanations (u) A. Penicillin is not indicated for treatment of an avulsed tooth. (h) B. Reimplantation is warranted as this is a permanent tooth with root intact. Primary teeth are never reimplanted. (u) C. See D for explanation. (c) D. Avulsed permanent teeth should be cleansed, transported in Hanks solution or saline and reimplanted by an oral surgeon within one hour. Ref: (29) 149. History & Physical/Endocrinology Which of the following is the most common presentation for an elderly female patient with primary hyperparathyroidism? A. Abdominal pain B. Renal lithiasis C. Acute pancreatitis D. Asymptomatic Explanations (u) A. Patients with primary hyperparathyroidism may have increased calcium levels which result in abdominal pain and constipation but this is not the most common manifestation. (u) B. Patients with primary hyperparathyroidism may have renal lithiasis as a result of the hypercalcemia but it is not the most common presentation. (u) C. Patients with acute pancreatitis may have hypocalcemia as a result of the calcium soaps that are formed in order to help to process the food that is eaten. It is not related to hyperparathyroidism; rather it is most commonly caused by gallstones and alcoholism. (c) D. Patients with primary hyperparathyroidism are most commonly found to have this disease by an incidental finding of hypercalcemia on routine laboratory testing as a result of screening. Up to 0.1% of the adult population has this condition which is most commonly seen in females over age 50. Ref: (28) 150. Diagnosis/Obstetrics/Gynecology A 35 year-old G2P1001 female presents to the office at 11 weeks gestation with vaginal bleeding, mid-lower abdominal cramping, and bilateral lower pelvic discomfort. On examination, bright red blood is seen coming from the cervical os. The cervix is closed. The uterus is 9-11 weeks in size by palpation. Her blood pressure is 120/70 mmHg and her pulse rate is 96. What is the patient's most likely diagnosis? A. Inevitable abortion B. Incomplete abortion C. Threatened abortion D. Missed abortion

57

Explanations (u) A. Inevitable abortion is when the patient presents during the first 20 weeks of pregnancy with bleeding and crampy abdominal pain, also associated with a dilated cervix or a gush of fluid without the passage of the products of conception. (u) B. Incomplete abortion occurs when part of the products of conception are passed from the cervix, the cervix will also be dilated. (c) C. If bleeding occurs in the first 20 weeks of pregnancy and the cervix is closed, threatened abortion is the diagnosis. (u) D. A missed abortion occurs when a patient presents with smaller gestational size by examination than by dates and no fetal heart tones. Ref: (4) 151. Diagnostic Studies/Dermatology A 23 year-old African American female presents with concern regarding "lightening" of the skin in her knees, face, clavicles, and wrists. These areas appear to be increasing in size. She was recently diagnosed with pernicious anemia. In order to confirm her diagnosis, which of the following tests is indicated? A. Wood's light B. KOH prep C. Skin biopsy D. Gram stain Explanations (u) A. Wood's light is primarily used to evaluate macules but there are no pathognomonic fluorescent patterns that would confirm the diagnosis of vitiligo. (u) B. KOH prep would be indicated to search for tinea versicolor by showing a spaghetti and meatball appearance but this patient's condition is consistent with vitiligo. (c) C. Skin biopsy in a patient with vitiligo will show complete absence of melanocytes and will confirm the diagnosis. (u) D. Gram stain is helpful in diseases secondary to a bacterial etiology not vitiligo. Ref: (10) 152. Clinical Therapeutics/Orthopedics/Rheumatology A 52 year-old male presents with a severely painful, swollen right great toe. He denies recent trauma, but reports several similar episodes of toe pain and swelling over the past two to three years. He has a history of alcohol abuse and hypertension, for which he "takes medication" of unknown type. Examination reveals bright erythema and edema associated with the right first MCP joint. Which of the patient's antihypertensive medications may be contributing to this condition? A. Diltiazem (Cardizem) B. Hydrochlorothiazide (HCTZ) C. Lisinopril (Prinivil) D. Metoprolol (Lopressor) Explanations (u) A. Calcium-channel blockers, such as diltiazem, are not associated with increased risk of hyperuricemia and gout. (c) B. Thiazide diuretics, such as hydrochlorothiazide, are associated with increased risk of hyperuricemia and gout. (u) C. ACE inhibitors, such as lisinopril, are not associated with increased risk of hyperuricemia and gout. (u) D. Beta-blockers, such as metoprolol, are not associated with increased risk of hyperuricemia and gout. Ref: (11) 153. Clinical Intervention/Cardiology A 66 year-old female with a history of coronary artery disease presents with a new onset of dizziness and fatigue for two weeks. She recalls nearly passing out on one occasion. Examination is unremarkable except for bradycardia. Electrocardiogram (ECG) reveals a heart rate of 50 with a normal PR interval followed by a normal QRS. There are several non-conducting P waves and no lengthening of the PR interval. Which of the following interventions is the therapy of choice?

58

A. Permanent pacemaker B. Radio-frequency ablation C. Maze procedure D. Automatic Implantable Cardioverter Defibrillator Explanations (c) A. This patient has symptomatic second degree type II heart block and requires a pacemaker. (u) B. Radio-frequency ablation is useful for supraventricular tachyarrhythmias. (u) C. Maze procedure is a surgical procedure for decreasing the incidence of atrial fibrillation by creating cuts simulating a pathway in the atriums. (u) D. Automatic Implantable Cardioverter Defibrillator (AICD) is used in patients at risk for sudden death. Ref: (11) 154. History & Physical/Neurology A mother brings in her 4 year-old son with complaints that he falls frequently and he "stands funny". The mother also notes that he has lost the ability to easily stand from a seated position. She reports that he met developmental milestones as an infant. Which of the following is the most likely cause? A. Developmental hip dysplasia B. Myasthenia gravis C. Cerebral palsy D. Muscular dystrophy

Explanations (u) A. Signs and symptoms of developmental hip dysplasia would have been present more at birth and when beginning to crawl and walk. (u) B. Myasthenia gravis (MG) is less likely due to age and MG typically affects the ocular, pharyngeal and respiratory muscles. (u) C. Cerebral palsy (CP) typically presents with ataxia, spasticity or tightness, and uncoordination. Patients with CP typically do not meet developmental milestones. (c) D. Muscular dystrophy age of onset is by age 5, and begins in the pelvic girdle. Ref: (13) 155. Clinical Therapeutics/Pulmonology Which of the following classes of medications is most likely to cause a persistent cough? A. Tricyclic antidepressants B. Calcium channel blockers C. Beta-adrenoceptor blocking agents D. Angiotensin converting enzyme inhibitors Explanations (u) A. Adverse effects of tricyclic antidepressants are due to blockage of acetylcholine receptors causing such symptoms as blurred vision, dry mouth, urinary retention and constipation. (u) B. Side effects are uncommon with calcium channel blockers, but include constipation, dizziness, headache and a feeling of fatigue. (u) C. Common adverse effects of beta-adrenoceptor blocking agents include bradycardia and central nervous system symptoms, such as fatigue or insomnia. (c) D. Common side effects of angiotensin converting enzyme inhibitors (ACE inhibitors) include a dry cough. Ref: (21) 156. Scientific Concepts/ENT/Ophthalmology Which of the following is the most likely organism in a 2 year-old child with acute otitis media? A. Staphylococcus aureus B. Moraxella catarrhalis C. Pseudomonas aeruginosa D. Streptococcus pneumoniae

59

Explanations (u) A. See D for explanation. (u) B. See D for explanation. (u) C. See D for explanation. (c) D. The most common pathogens in children with acute otitis media are Streptococcus pneumoniae, Haemophilus influenzae, and Streptococcus pyogenes. Ref: (28) 157. Diagnosis/Urology/Renal A 15 year-old male patient presents with oliguria, hematuria, proteinuria, and fatigue following streptococcal pharyngitis 2 weeks ago. Which of the following is the most likely diagnosis? A. Acute pyelonephritis B. Acute glomerulonephritis C. Systemic lupus erythematosus D. Initial onset of type 1 diabetes mellitus Explanations (u) A. Acute pyelonephritis presents with fever, flank pain, urgency, and pyuria. (c) B. Acute glomerulonephritis is a complication that can follow a streptococcal infection after 1 to 3 weeks. (u) C. Systemic lupus erythematosus is not a complication of streptococcal infections, but it can cause rapid progressive glomerulonephritis. (u) D. Diabetes mellitus presents with polyuria, not oliguria or hematuria. Ref: (28) 158. Diagnostic Studies/Cardiology Pulmonary capillary wedge pressure indirectly measures which of the following? A. Right ventricular end-diastolic pressure B. Right atrial filling pressure C. Left ventricular systolic pressure D. Left atrial filling pressure Explanations (u) A. See D for explanation. (u) B. See D for explanation. (u) C. See D for explanation. (c) D. Pulmonary capillary wedge pressure indirectly measures left atrial filling pressures. Ref: (30) 159. Clinical Therapeutics/Endocrinology Which of the following is a known complication to prescribing excessive doses of levothyroxine (Synthroid) for patients with hypothyroidism? A. Primary hyperparathyroidism B. Papillary carcinoma of the thyroid gland C. Osteoporosis D. Diabetes Insipidus Explanations (u) A. Primary hyperparathyroidism is not related to excessive doses of levothyroxine. (u) B. Papillary thyroid cancer is the most common thyroid cancer. It is most closely associated with radiation exposure. Levothyroxine is typically given after surgical treatment in order to suppress any thyroid activity in any remaining thyroid tissue. (c) C. Osteoporosis may result from overaggressive therapy with levothyroxine (Synthroid) because of the increased bone turnover that results from increased basal metabolic rate. (u) D. Diabetes insipidus results from lack of production of ADH in the brain or from the kidney not responding to ADH in the circulation. It is not related to levothyroxine therapy. Ref: (28)

60

160. History & Physical/Psychiatry/Behavioral Medicine A patient tells you that he is receiving special messages from the TV every night at 7:00 pm. This is an example of which of the following? A. Delusions B. Ideas of reference C. Paranoia D. Suicidal ideation Explanations (u) A. Delusions are false fixed beliefs. (c) B. Ideas of reference are fixed beliefs that people are referring to you and about you through media. (u) C. He is frankly telling you this and not displaying anything that makes you think he is paranoid. (u) D. He is not stating he is having thoughts of harming or killing himself. Ref: (14) 161. Clinical Intervention/Orthopedics/Rheumatology A 22 year-old male presents several hours after sustaining a hand injury when he punched a wall. X-rays of the hand demonstrate fracture of the fifth metacarpal neck with 65 degrees dorsal angulation and a claw hand. What is the most appropriate intervention? A. Antibiotic treatment and ulnar gutter splint immobilization B. Closed reduction and ulnar gutter splint immobilization C. Open reduction and ulnar gutter splint immobilization D. Ulnar gutter splint immobilization only Explanations (u) A. Antibiotic treatment would be appropriate if the skin was broken and the injury was sustained in a fist fight with the potential for introduced oral flora. (u) B. Reduction followed by splinting is recommended for fifth metatarsal fractures with angulation of 15-40 degrees. (c) C. Open reduction is indicated with angulation of greater than 40 degrees. (u) D. With angulation of greater than 15 degrees, reduction should be performed prior to splinting. Ref: (29) 162. Diagnosis/Gastrointestinal/Nutritional A male patient presents for a routine physical examination. He denies chronic health problems, regular medication use or previous surgeries. He exercises 4-5 times weekly, does not use tobacco products, and consumes alcohol in moderation. His last intake of alcohol was two weeks ago while on a trip to Mexico. His review of systems and physical examination are both negative. Routine labs were drawn which were significant for the following: Total serum bilirubin 3.5 mg/dL (0.3-1.0 mg/dL), direct bilirubin 0.2 mg/dL (0.1-0.3 mg/dL), AST 35 U/L (0-35 U/L), ALT 30 U/L (0-35 U/L), Alkaline Phosphatase 48 U/L (30-120 U/L) and GGT 12 U/L (1-94 U/L). What is the most likely diagnosis in this patient? A. Alcoholic hepatitis B. Crigler-Najjar syndrome C. Gilbert's syndrome D. Wilson's disease Explanations (u) A. Alcoholic hepatitis presents with elevated ALT and AST. (u) B. The Crigler-Najjar syndrome is typified by an isolated elevation in the unconjugated serum bilirubin and usually causes the bilirubin to be elevated at higher levels (6-45 mg/dL). (c) C. Gilbert's syndrome is a relatively common cause of mild isolated elevations in indirect serum bilirubin. (u) D. Wilson's disease is a disorder of impaired biliary copper excretion which among many other problems will cause a rise in total serum bilirubin but also significant elevations in AST. Ref: (7)

61

163. Health Maintenance/ENT/Ophthalmology Which of the following is the leading cause of permanent visual loss in a patient over the age of 75? A. Blepharitis B. Cataracts C. Central retinal artery occlusion D. Macular degeneration Explanations (u) A. Blepharitis is a chronic bilateral inflammatory condition of the lid margins. (u) B. Cataracts are the clouding of the lens sufficient to reduce vision. Most develop slowly as a result of aging, leading to gradual impairment of vision. (u) C. Central retinal artery occlusion presents as a rare cause of sudden profound monocular visual loss. (c) D. Age-related macular degeneration is the leading cause of permanent visual loss in the older population. The exact cause is unknown, but the prevalence increases with each decade over age 50 years. Ref: (28) 164. Diagnostic Studies/Pulmonology A 42 year-old male smoker presents for further evaluation of a 4 cm solitary pulmonary nodule discovered on a recent chest x-ray. Which of the following diagnostic tests is indicated next? A. Bone scan B. Thoractotomy C. Mediastinoscopy D. CT scan of chest Explanations (u) A. Bone scanning is indicated for evaluation of bone metastases, most commonly secondary to cancer of the breast or prostate. (u) B. Diagnostic thoractotomy is indicated for biopsy of the lesion should the CT scan of the chest indicate a suspicious malignant lesion that is inaccessible to thoracoscopy. (u) C. Mediastinoscopy can be utilized to further evaluate any enlarged mediastinal lymph nodes that may be found on the CT scan of the chest, but is not indicated prior to the CT scan. (c) D. A CT scan of the chest is needed to further evaluate the characteristics of the solitary pulmonary nodule and to determine lymph node involvement or presence of multiple lesions. Ref: (7) 165. Clinical Therapeutics/Infectious Diseases A 16 year-old female who is a competitive swimmer presents with a complaint that her toe nails are thickened and discolored. Examination reveals that the nails have a dry yellow brittle appearance along with a friable surface of the nail plate. Which of the following is the treatment of choice? A. Metronidazole (Flagyl) B. Diflucan C. Terbinafine (Sporonox) D. Acyclovir Explanations (u) A. Metronidazole is not indicated for fungal infections of the nails. (u) B. Diflucan is indicated for dermal fungal infections. (c) C. Oral Terbinafine 250 mg is considered the best treatment for onychomycosis. (u) D. Acyclovir is indicated for viral infections, not fungal infections. Ref: (28)

62

166. History & Physical/Cardiology A 26 year-old male presents with increased dyspnea with exercise. He has noted a decrease in his exercise tolerance over the past several months. He denies chest pain or skipped heart beats. Echocardiogram reveals left ventricular hypertrophy with asymmetric septal hypertrophy. Ejection fraction is 65%. Which of the following is the most likely presenting history or physical exam finding? A. He has an older brother with the same diagnosis. B. An S3 gallop is heard. C. Patient notes completing a course of adriamycin. D. Elevated jugular venous distension is noted. Explanations (c) A. Hypertrophic cardiomyopathy can be genetic and present in 25% of first degree relatives. (u) B. Dilated cardiomyopathy may present with an S3 gallop due to volume overload. (u) C. Adriamycin chemotherapy can lead to cardiac dysfunction, dilated cardiomyopathy and eventually heart failure. (u) D. Restrictive or dilated cardiomyopathy may present with JVD due to abnormalities of filling. Ref: (11) 167. Clinical Intervention/Neurology A 23 year-old patient presents with two days of fatigue, headache, fever and pain around the area in which she was bitten by a stray baby raccoon in an unprovoked attack 10 days ago. She cleaned the small wound thoroughly. Which of the following is the most appropriate intervention in this patient? A. Do nothing and treat her symptoms B. Human diploid cell rabies vaccine, 5 injections given all at once now C. Rabies immunoglobulin only D. Rabies immunoglobulins and human diploid cell rabies vaccine given 5 times in a 1-month period Explanations (u) A. See D for explanation. (u) B. See D for explanation. (u) C. See D for explanation. (c) D. Bites by bats, skunks, and raccoons always require this regimen if the animal is not caught and tested. Ref: (28) 168. Diagnosis/Dermatology A 55 year-old female presents with a lesion on her face that is painful, bright red, with distinct raised borders. She also is complaining of fever and chills. There is no evidence of any marks which would show a portal of entry. Which of the following is the most likely diagnosis? A. Impetigo B. Erythrasma C. Cellulitis D. Erysipelas Explanations (u) A. Impetigo is a bacterial infection of the skin that is associated with honey-colored crusts. (u) B. Erythrasma is a bacterial infection of the skin that is red in appearance and fluoresces a coral red color under a Wood's lamp. Systemic symptoms are rarely seen and the borders tend to be less distinct. (u) C. Cellulitis has many features of erysipelas but it goes on to involve the subcutaneous tissues. Patients with cellulitis will not have raised borders and demarkation from uninvolved skin is not distinct with this condition. (c) D. Erysipelas is a distinct type of superficial cellulitis with redness, a distinct and raised border, and sharp demarkation from uninvolved skin. It is typically associated with systemic symptoms such as fever and chills. It is caused by group A strep most commonly. Ref: (10)

63

169. Diagnostic Studies/Obstetrics/Gynecology What should be the initial evaluation of the etiology of infertility in a 25 year-old female who has been trying to conceive for 1 year? A. Endometrial biopsy B. Hysterosalpingogram C. Serum progesterone levels D. Basal body temperature measurements Explanations (u) A. Endometrial biopsy will identify changes associated with infertility, however it is invasive and often done later in the evaluation. (u) B. Hysterosalpingogram provides information about the internal female genital tract. This is usually done later in the work-up. (u) C. Serum progesterone levels drawn at mid-luteal phase will help determine normal from abnormal cycles. Levels above 15 ng/mL will indicate a normal cycle. (c) D. Basal body temperature measurement is an excellent screening test for ovulation, and will help with the timing of coitus. Ref: (4) 170. Scientific Concepts/Gastrointestinal/Nutritional What is the pathologic mechanism of Hirschsprung's disease? A. Pyloric circular muscle hypertrophy causing gastric outlet obstruction B. Absence of ganglion cells in the mucosal and muscular layers of the colon C. A defect in the diaphragm leading to protrusion of the abdominal viscera into the thoracic cavity D. Absence of relaxation of the lower esophageal sphincter and lack of peristalsis in the esophageal body Explanations (u) A. Pyloric circular muscle hypertrophy causing gastric outlet obstruction describes pyloric stenosis. (c) B. Hirschsprung's disease results from an absence of ganglion cells in the mucosal and muscular layers of the colon. (u) C. A defect in the diaphragm leading to protrusion of the abdominal viscera into the thoracic cavity describes congenital diaphragmatic hernia. (u) D. Absence of relaxation of the lower esophageal sphincter and lack of peristalsis in the esophageal body describes achalasia of the esophagus. Ref: (13) 171. History & Physical/ENT/Ophthalmology Which of the following is described as a harmless triangular nodule in the bulbar conjunctiva on either side of the iris? A. Corneal arcus B. Hordeolum C. Pinguecula D. Xanthelasma Explanations (u) A. Corneal arcus is an extremely common, bilateral, benign peripheral corneal degeneration, associated hyperlipidemia. (u) B. Hordeolum is characterized by a localized red, swollen, acutely tender area on the upper or lower lid. (c) C. Pinguecula is a yellow elevated conjunctival nodule, more commonly on the nasal side, in the area of the palpebral fissure. (u) D. Xanthelasma appear as yellow plaques that occur on the anterior surface of the eyelid, usually bilaterally near the inner angle of the eye. Ref: (3)

64

172. Clinical Therapeutics/Pulmonology A 2 year-old presents to the emergency department in acute respiratory distress. The parents relate a history of a recent upper respiratory illness that was followed by a sudden onset of barking cough during the night, but this morning they noted increased difficulty breathing. The child is noted to have stridor at rest, but has no evidence of cyanosis. Which of the following is the most appropriate initial intervention? A. Intravenous antibiotics B. Endotracheal intubation C. Inhaled mucolytic agent D. Nebulized racemic epinephrine Explanations (u) A. Laryngotracheobronchitis is caused by viruses, not bacteria, and therefore antibiotic therapy is not indicated. (u) B. If patients fail to respond to initial treatment and progress to impending respiratory failure, endotracheal intubation is then indicated. (u) C. Inhaled mucolytic agents are not indicated in the treatment of laryngotracheobronchitis. (c) D. This patient most likely has laryngotracheobronchitis (viral croup). Treatment with nebulized racemic epinephrine and glucocorticosteroids is indicated for patients with stridor at rest. Ref: (13) 173. Health Maintenance/Cardiology Which of the following lifestyle recommendations would most benefit the heart failure patient's quality of life? A. Begin a regular exercise program B. Total salt restriction C. Home monitoring of blood pressure D. Increase home oxygen requirements Explanations (c) A. Exercise training improves activity tolerance and deconditioning offering some recompensation. (u) B. Although salt restriction is a recommendation total salt restriction would be unrealistic to achieve. (u) C. Home monitoring of blood pressure will not improve a heart failure patient's quality of life. (u) D. Increasing the requirements for home oxygen could signal that they are worsening and is palliative for their quality of life. Ref: (28) 174. Diagnosis/Obstetrics/Gynecology On rectovaginal examination of a 72 year-old post-menopausal female a 3 cm by 3 cm right adnexal mass is palpated. The rest of her physical examination is unremarkable. Her last gynecological examination was last year and was unremarkable. Which of the following is the most likely diagnosis? A. Ovarian cancer B. Endometrioma C. Uterine cancer D. Functional ovarian cyst Explanations (c) A. Ovaries should not be palpable in the postmenopausal patient. Consider ovarian cancer until proven otherwise. (u) B. An endometrioma is an endometriosis cyst of the ovary. The cyst is filled with thick, chocolate-colored fluid, often called a "chocolate cyst". These cysts occur in menstruating women. (u) C. 90% of patients with endometrial cancer present with vaginal bleeding. The uterus may be enlarged on examination, however, a palpable ovary is not associated with this condition. (u) D. Functional ovarian cysts occur in women who are ovulating. Ref: (4)

65

175. Clinical Intervention/Gastrointestinal/Nutritional A pregnant female presents at 32 weeks gestation with painless rectal bleeding and a bulging perianal mass when straining which goes away. Which of the following is the most appropriate management of this patient? A. Hemorrhoidectomy B. Metronidazole (Flagyl) C. Psyllium (Metamucil) D. Sclerotherapy Explanations (u) A. Hemorrhoidectomy and sclerotherapy are reserved for severe Grade III and IV hemorrhoids. (u) B. Metronidazole is not indicated in the treatment of hemorrhoids. (c) C. Dietary fiber or psyllium bulk laxatives can be used to decrease straining with defecation. (u) D. See A for explanation. Ref: (28) 176. Diagnostic Studies/Neurology A 12 month-old in the emergency department is diagnosed with possible viral meningitis. Which of the following cerebral spinal fluid (CSF) laboratory results is most consistent with this diagnosis? A. Decreased CSF glucose level and increased protein B. Decreased CSF total protein level and very few neutrophils C. Increased CSF mononuclear cells and normal glucose D. Increased CSF C-reactive protein and normal glucose

Explanations (u) A. Decreased CSF glucose level and increased protein is consistent with bacterial meningitis. (u) B. Decreased CSF total protein level and very few neutrophils is a finding in syphilitic meningitis. (c) C. In aseptic meningitis, CSF shows mainly mononuclear cells within 6-8 hours, glucose is normal and there is normal to lower protein. (u) D. Increased CSF C-reactive protein and normal glucose is found in any inflammatory process affecting the CSF. Ref: (1) 177. Scientific Concepts/Psychiatry/Behavioral Medicine Which of the following medical complications of eating disorders is due to purging (vomiting/laxative abuse)? A. Intestinal obstruction B. Reduced thyroid metabolism C. Amenorrhea D. Electrolyte abnormalities Explanations (u) A. Intestinal obstruction is not caused by vomiting. (u) B. Reduced thyroid metabolism is a complication of anorexia nervosa related to cachexia. (u) C. Amenorrhea occurs as a result of anorexia nervosa with weight loss. (c) D. Electrolyte abnormalities, particularly hypokalemia, hypochloremic alkalosis, and hypomagnesemia may occur as a result of purging. Ref: (14) 178. Clinical Therapeutics/Dermatology Which of the following is considered the antibiotic of choice in the treatment of human bite wounds? A. Ampicillin B. Penicillin C. Augmentin D. Ciprofloxacin

66

Explanations (u) A. See C for explanation. (u) B. See C for explanation (c) C. Augmentin is considered to be the antibiotic of choice for human bites that may be contaminated with Eikenella corrodens, strep viridans, and staph aureus. (u) D. See C for explanation. Ref: (7) 179. History & Physical/Cardiology A newborn presents with blue discoloration of the peripheral extremities which worsens with exposure to cold and improves with warming. Which of the following is the most likely cause? A. Raynauds phenomenon B. Livedo reticularis C. Erythromelagia D. Acrocyanosis Explanations (u) A. Raynauds phenomenon has a triphasic color response and is exacerbated by cold or emotions. (u) B. Livedo reticularis is characterized by a lacy pattern on the skin of the lower extremities. (u) C. Erythromelagia is red, painful extremities. (c) D. Acrocyanosis is a blue discoloration of the digits, usually of the peripheral extremities in newborns, which worsens with exposure to cold and improves with warming. Ref: (11) 180. Diagnosis/Urology/Renal A 25 year-old female presents with right lower quadrant pain, right flank pain, nausea, and vomiting. Her temperature is 39.6 degrees C. There is right CVA tenderness and RLQ tenderness. Pelvic exam is unremarkable. Urinalysis reveals pH 7.0, trace protein, negative glucose, negative ketones, positive blood, and positive nitrates. Specific gravity is 1.022. Microscopic shows 102 RBCs/HPF, 50-75 WBCs/HPF, rare epithelial cells, and WBC casts. The most likely diagnosis is A. acute salpingitis. B. nephrolithiasis. C. acute pyelonephritis. D. appendicitis. Explanations (u) A. Acute salpingitis would be suggested if pelvic exam abnormalities were present. (u) B. Nephrolithiasis does not usually present with fever or casts. Urinalysis will have RBCs present. (c) C. Acute pyelonephritis presents with flank pain, fever, and generalized muscle tenderness. Urinalysis shows pyuria with leukocyte casts. (u) D. This scenario is consistent with acute pyelonephritis, not acute appendicitis. Ref: (28) 181. Clinical Intervention/Pulmonology A 62 year-old male presents with a right hilar mass. Needle-biopsy of the mass reveals the presence of small-cell carcinoma and a bone scan reveals the presence of scattered hot spots throughout the skeleton. Which of the following is the most appropriate treatment? A. Lobectomy B. Pneumonectomy C. Thoracic radiation therapy D. Combination chemotherapy

67

Explanations (u) A. Small-cell carcinoma of the lung is rarely treatable with surgical resection. Surgery may be indicated as part of the treatment protocol for small peripheral lesions without any evidence of metastasis. (u) B. See A for explanation. (u) C. While thoracic radiation therapy has been shown to be beneficial for patients with limited small-cell lung cancer, no benefit has been observed for patients with extensive disease defined as the presence of metastatic disease. (c) D. Combination chemotherapy is the treatment of choice for a patient with small-cell carcinoma of the lung. Ref: (28) 182. Diagnostic Studies/Gastrointestinal/Nutritional A patient presents with a long history of dyspepsia refractory to maximum appropriate therapy. A recent upper GI series revealed multiple gastric and duodenal ulcerations with prominent mucosal folds. What diagnostic study would be confirmatory of this patient's suspected diagnosis? A. EGD with duodenal biopsy B. Serum gastrin level C. Helicobacter pylori serology D. Abdominal CT scan Explanations (u) A. An endoscopic biopsy of the duodenum may be necessary in some cases to ensure the absence of cancer, etc., but a biopsy in this area is not confirmatory for Zollinger-Ellison Syndrome, which is highly suspect, nor does it allow samples of the gastric mucosa to be studied. (c) B. Serum gastrin levels can be elevated for many reasons; however significant elevations in a fasting state as well as with the secretin stimulation test are confirmatory for Zollinger-Ellison syndrome which is highly suspect in this case. (u) C. Helicobacter pylori can cause symptoms refractory to medical management and an infection can occur concomitantly with Zollinger-Ellison syndrome, but it does not confirm the diagnosis of a gastrinoma or similar hypersecretory disease. (u) D. An abdominal CT scan is not specific enough to confirm the diagnosis of Zollinger-Ellison syndrome. Ref: (7) 183. Scientific Concepts/ENT/Ophthalmology What is the most common location of anterior nasal epistaxis? A. Middle turbinate B. Posterior ethmoid artery C. Kiesselbach's triangle D. Inferior turbinate Explanations (u) A. See D for explanation. (u) B. See D for explanation. (c) C. 90% of all cases of anterior epistaxis originate from Kiesselbach's triangle. (u) D. The inferior turbinate is the most common location for posterior epistaxis. Ref: (29) 184. Diagnosis/Hematology A 48 year-old male complains of weakness and general malaise for about 2 months. Patient denies any recent illness and does not take any medications. Physical examination reveals a pale looking male in no acute distress. His heart rate is 110 beats/minute without a murmur and his abdominal examination reveals hepatosplenomegaly. A CBC reveals the WBC to be 62,000/microliter, Hgb is 8.3 gms/dl, Hct is 24.6%. A differential reveals a predominance of monoblasts and promyelocytes with Auer rods present. What is the most likely diagnosis? A. Acute lymphocytic leukemia B. Acute myelogenous leukemia C. Chronic lymphocytic leukemia D. Chronic myelogenous leukemia

68

Explanations (u) A. Acute lymphocytic leukemia (ALL) is a leukemia most often seen in children with lymphoblasts predominating. Adults who develop ALL usually have a prolymphocytic cell presentation. (c) B. Acute myelogenous leukemia (AML) is a leukemia that presents in adults with a cell lineage of the immature granulocytic cells seen in the peripheral blood e.g. myeloblasts, promyelocytes. Auer rods are commonly seen in this condition. (u) C. Chronic lymphocytic leukemia (CLL) is a leukemia that presents in older adults with WBC counts up to 100,000/mm3. On peripheral blood smears the cell that predominates is a mature lymphocyte. (u) D. Chronic myelogenous leukemia (CML) is a leukemia that presents in adults with a cell lineage of mature cell lines with a marked increase in basophils and eosinophils. These cells are also known for having the Philadelphia chromosome. Ref: (28) 185. Clinical Therapeutics/Obstetrics/Gynecology A 56 year-old female patient comes to the office for evaluation. She complains of dyspareunia and a thin vaginal discharge. On physical examination atrophic vulvar changes are noted associated with vaginal petechiae and a thin clear discharge. What medication is recommended to treat this patient's symptoms? A. Topical estrogen (Estrace) cream B. Topical hydrocortisone (Gynecort) C. Metronidazole (MetroGel) vaginal gel D. Terconazole (Terazol) vaginal suppository Explanations (c) A. This patient has atrophic vaginitis which is very common in postmenopausal patients. The vagina will appear atrophied, will look pale, and thin and dry in appearance. Best to treat with topical or oral estrogen preparations. (u) B. Topical hydrocortisone would not be used in atrophic vaginitis. (u) C. MetroGel is the drug of choice for bacterial vaginosis. (u) D. Terazol is the treatment for non-albicans yeast infections. Ref: (4) 186. History & Physical/Orthopedics/Rheumatology The neuromuscular hallmark of polymyalgia rheumatica is A. asymmetric muscle pain and stiffness. B. distal paresthesias. C. muscle weakness. D. proximal muscle pain and stiffness. Explanations (u) A. Polymyalgia rheumatica is associated with pain and stiffness but typically occurs in a symmetric distribution. (u) B. Peripheral neurons are unaffected by polymyalgia rheumatica and is not associated with neuropathic symptoms. (u) C. Although patients may appear to have proximal muscle weakness, this is usually related to pain and is not true measurable weakness. (c) D. Proximal symmetric muscle pain and stiffness, particularly involving the shoulder, neck and pelvic girdle, is the musculoskeletal hallmark of polymyalgia rheumatica. Ref: (11) 187. Diagnosis/Cardiology A two month-old infant appeared well until three weeks ago when he became dyspneic and had difficulty feeding. A 4/6 holosystolic murmur is heard at the left lower sternal border in the 3rd ICS. An electrocardiogram (ECG) shows left and right ventricular hypertrophy. Which of the following is the most likely diagnosis? A. Atrial septal defect B. Pulmonary hypertension C. Ventricular septal defect D. Tricuspid insufficiency

69

Explanations (u) A. An ASD usually presents with a soft mid-systolic murmur in the second left ICS with a widely split and fixed S2. (u) B. With pulmonary hypertension the murmur may be most consistent with pulmonary or tricuspid insufficiency. (c) C. This is a classic presentation for a ventricular septal defect. (u) D. Tricuspid insufficiency is a systolic murmur heard best at the 4th ICS that may radiate to the apex. Ref: (11) 188. Diagnostic Studies/Pulmonology A 56 year-old male with a 40 pack-year smoking history presents complaining of progressive shortness of breath. Spirometry reveals an FEV1 of 2 L (40% of predicted), an FVC of 4 L (80% of predicted) and an FEV1/FVC of 50%. These findings are most consistent with A. sarcoidosis. B. chronic bronchitis. C. interstitial lung disease. D. congestive heart failure. Explanations (u) A. Sarcoidosis, interstitial lung disease and congestive heart failure most commonly produce a restrictive pattern on spirometry with a reduction in forced expiratory volume in one second (FEV1) and forced vital capacity (FVC) secondary to the decrease in total lung capacity (TLC), but the FEV1/FVC ratio is usually normal or increased, not decreased. (c) B. The reduced FEV1 and FEV1/FVC is characteristic of an obstructive pattern that is seen in chronic obstructive pulmonary diseases, such as chronic bronchitis. (u) C. See A for explanation. (u) D. See A for explanation. Ref: (1) 189. Clinical Intervention/Dermatology An elderly woman presents to your clinic complaining of unilateral facial pain and painful lesions. She also complains of blurred vision in the ipsilateral eye. On examination she has several vesicles on an erythematous base, some of the lesions with crusts. They are distributed in a dermatomal pattern and involve the skin overlying the maxillary region and the tip of her nose. Which of the following is the next most appropriate intervention in the care of this patient? A. KOH prep B. Culture for bacteria C. Referral to an ophthalmologist D. Application of corticosteroids Explanations (u) A. A KOH prep examining for fungal elements is not indicated in this case. (u) B. While bacterial infection may occur along with the viral infection, a bacterial culture is not usually employed. (c) C. Immediate referral to an ophthalmologist is needed when herpes keratitis is suspected, as in this case. A fluorescein stain of the eye might reveal the typical dendritic corneal lesion. (h) D. Application of corticosteroids may cause proliferation of the virus and should only be used by an ophthalmologist. Ref: (28) 190. Diagnosis/Endocrinology A male is found to have an eunuchoid body habitus, gynecomastia, diminished secondary sex characteristics, and small and firm testicles. Which of the following is the most likely diagnosis? A. Fragile X syndrome B. Gaucher disease C. Klinefelter syndrome D. Kallmann syndrome

70

Explanations (u) A. Fragile X syndrome is the second most common inherited form of mental retardation. Affected males show macroorchidism (enlarged testis) after puberty, large ears, and a prominent jaw, high-pitched voice, and mental retardation. (u) B. Patients with Gaucher disease have anemia and thrombocytopenia as its major manifestation. These patients may also have cortical erosions of bone and may have bone pain due to local infarctions. (c) C. Men with Klinefelter syndrome have an extra X chromosome and are phenotypically normal until puberty. After puberty, these men have long legs and arms, a female escutcheon, gynecomastia, and small testes. Low serum testosterone and infertility along with a decrease in executive functioning are common in these men. (u) D. Kallmann's syndrome is the most common cause of congenital isolated gonadotropin deficiency. It is associated with anosmia and hyposmia caused by hypoplasia of the olfactory bulbs. About one-half of these patients will have renal agenesis. There is also an increase in cryptorchism, sensorineural hearing deafness, and cleft lip. Ref: (28) 191. History & Physical/Gastrointestinal/Nutritional Which of the following is best described as a hard nodule in the periumbilical region? A. Hodgkin's lymphoma B. Gastric bezoar C. Sister Mary Joseph nodule D. Virchow's node Explanations (u) A. Hodgkin's lymphoma will more likely affect intra-abdominal lymph nodes. (u) B. Gastric bezoar can present as a palpable abdominal mass, but is not indicative of intra-abdominal/intra-pelvic cancer. (c) C. The Sister Mary Joseph nodule can occur with metastatic gastric and pancreatic cancers. The nodule is generally minimally to non-tender to palpation. (u) D. Virchow's node is found in cases of metastatic disease, but is represented by a left supraclavicular lymph node. Ref: (7) 192. Diagnostic Studies/Cardiology Which of the following laboratory tests would be elevated in a patient with symptomatic heart failure? A. Lactate dehydrogenase (LDH) B. Troponin I (TnI) C. C-reactive protein (CRP) D. Brain natriuretic peptide (BNP) Explanations (u) A. LDH is not a sensitive marker in patients with heart failure. (u) B. Troponin measurements are specific for myocardial infarctions. (u) C. Increased serum levels of CRP are found in patients with unstable angina and MI. They can be a strong predictor of coronary events. (c) D. BNP is expressed in the ventricles and is a sensitive assay in patients with heart failure. Ref: (28) 193. Clinical Therapeutics/ENT/Ophthalmology A 34 year-old female presents with recurrent bouts of dizziness, tinnitus, and hearing loss. She states that the episodes are incapacitating and cause her to become nauseous and vomit. The attacks last about one hour and the symptoms disappear after a few days. The last two episodes were treated with meclizine (Antivert) and prochlorperazine (Compazine) at the emergency room. Audiologic testing reveals low-tone frequency hearing loss. Which of the following is the most appropriate long-term management for this patient? A. Epley maneuver B. Diuretics and low-sodium diet C. Broad-spectrum antibiotics and Ibuprofen D. Scopolamine transdermal patch

71

Explanations (u) A. Epley maneuver is used to treat benign paroxysmal positional vertigo. (c) B. Diuretics and a low sodium diet will decreases the endolymphatic pressure in the semicircular canals, which is believed to be elevated in Meniere's disease, and help relieve symptoms. (u) C. Broad-spectrum antibiotics and ibuprofen are used to treat otitis media, not Meniere's disease. (u) D. A scopolamine patch is useful for treatment of a single episode, but not long-term management. Ref: (28) 194. Scientific Concepts/Pulmonology A 3 year-old male with cystic fibrosis develops pneumonia. Which of the following is the most likely etiology of the pneumonia? A. Escherichia coli B. Staphylococcus epidermidis C. Pseudomonas aueroginosa D. Streptococcus pneumoniae Explanations (u) A. See C for explanation. (u) B. See C for explanation. (c) C. Initially in the first few months of life, respiratory infection is common with Staphylococcus aureus and Haemophilus influenzae, but after that Pseudomonas aueroginosa becomes the major causative organism for infections. (u) D. See C for explanation. Ref: (13) 195. Clinical Intervention/Urology/Renal A 40 year-old patient with a history of recurrent kidney stones presents with acute onset of right flank pain and hematuria. The patient is afebrile and pain is poorly controlled on oral medications. On CT scan a 1 cm stone is noted in the renal pelvis. Which of the following is the most appropriate intervention for this patient? A. Antibiotics B. Shock wave lithotripsy C. Ureterolithotomy D. Fluid hydration Explanations (u) A. There is no indication of infection and antibiotics are not warranted at this time. (c) B. Extracorporeal shock wave lithotripsy is indicated in patients with stones greater than 6 mm in size or intractable pain. (u) C. Ureterolithotomy is an open surgical procedure and therefore very invasive. In this situation an ESWL would be as effective and safer. (u) D. While fluid hydration is indicated, a stone greater than 6 mm will typically not spontaneously pass and surgical therapy is indicated. Ref: (11) 196. Diagnosis/Infectious Diseases An unimmunized child has had a mild fever for several days. Today, the child is brought to the clinic because of the development of a rash. The rash is a pink discrete macular eruption mostly on the face and trunk. Postauricular and suboccipital lymph nodes are palpable. The child has a slight fever, but does not appear sick. Which of the following is the most likely diagnosis? A. Roseola B. Rubeola C. Rubella D. Erythema infectiosum

72

Explanations (u) A. Continued fever after rash onset and lymphadenopathy are not consistent with roseola. (u) B. High fever and lethargy are prominent features of rubeola and help distinguish it from rubella. (c) C. The characteristic lymphadenopathy and lack of systemic symptoms are most consistent with rubella. (u) D. The presence of lymphadenopathy is not consistent with erythema infectiosum, which typically presents with reddened cheeks. Ref: (28) 197. Diagnostic Studies/Orthopedics/Rheumatology A 74 year-old male presents with one month history of right shoulder pain without any known precipitant. His pain involves an area from the right paraspinous musculature to the right deltoid with occasional radiation down the arm. Pain is worse with movement of the shoulder and is not relieved by acetaminophen. He reports numbness of the right index finger and thumb. Physical examination of the shoulder is limited by pain. There is decreased grip strength and absent pinprick sensation in the index finger and thumb. Relexes are normal. What is the most appropriate initial study to obtain? A. EMG with nerve conduction B. MRI of the right shoulder C. Radiographs of the neck D. Radiographs of the right shoulder Explanations (u) A. Although electromyography and nerve conduction studies would be helpful in localizing the nerves involved, this is not an appropriate first-line diagnostic test. (u) B. Although there is pain in the shoulder, this patient presents with cervical radiculopathy and shoulder studies are not appropriate. (c) C. Cervical radiculopathy may be due to intervertebral foraminal osteophytes which will be identified on plain film radiographs of the neck making this the appropriate first step in evaluation. (u) D. Although there is pain in the shoulder, this patient presents with cervical radiculopathy and shoulder studies are not appropriate. Ref: (26) 198. Clinical Therapeutics/Cardiology When prescribing loop diuretics which of the following electrolytes should be most closely monitored? A. Calcium B. Potassium C. Sodium D. Chloride Explanations (u) A. See B for explanation. (c) B. Potassium along with magnesium should be monitored when prescribing loop diuretics. (u) C. See B for explanation. (u) D. See B for explanation. Ref: (11) 199. Diagnosis/Neurology An 8 year-old male presents with episodes described as 20-second lapses of awareness during which he blinks his eyes. After these attacks, he resumes his previous activity. Which of the following is the most likely diagnosis? A. Tonic-clonic seizure B. Myoclonic seizure C. Absence seizure D. Focal seizure

73

Explanations (u) A. Tonic-clonic seizures are associated with a loss of consciousness, tonic-clonic movements, and postictal confusion or sleep. (u) B. Myoclonic seizures are associated with mild myoclonic jerks of the neck and shoulder flexor muscles when falling asleep. (c) C. Absence seizures are characterized by lapses of consciousness associated with postictal confusion. (u) D. Focal seizures are associated with motor or sensory symptoms involving any portion of the body. Ref: (5) 200. History & Physical/Obstetrics/Gynecology You are performing a routine gynecological examination on a 49 year-old female. She states that for the last six months her periods have been getting heavier and lasting for seven days duration. She also tells you that she has been experiencing urinary frequency and constipation. Her abdominal exam is unremarkable. On pelvic examination you feel a slightly enlarged uterus and a left adnexal mass that moves with the uterine fundus. Which of the following is the most likely diagnosis? A. Adenomyosis B. Uterine fibroids C. Ovarian neoplasm D. Diverticular disease in the colon Explanations (u) A. Adenomyosis occurs when endometrial glands and stroma grow into the muscle of the uterus. Patients often have severe secondary dysmenorrhea and menorrhagia; the uterus will be symmetrically enlarged and often "boggy". (c) B. Patients with uterine fibroids often have menorrhagia and/ or metrorrhagia. On examination, the uterus will be larger, and irregularly-shaped. If a mass is present and it moves with the uterus, it is suggestive of a fibroid. (u) C. It is important to consider an ovarian neoplasm in a patient with a palpable adnexal mass, however, most adnexal masses are asymptomatic and not associated with menstrual changes. (u) D. Left-sided bowel disease is more common in older patients and would not be associated with menstrual changes. Ref: (4) 201. Diagnostic Studies/Pulmonology Which of the following findings confirm the adequacy of a sputum specimen for Gram stain and culture? A. Decreased red blood cells B. Decreased bronchial epithelial cells C. Increased Gram positive cocci D. Increased polymorphonuclear leukocytes Explanations (u) A. The presence or absence of red blood cells is not a criterion for determining the adequacy of a sputum sample. (u) B. The presence of bronchial epithelial cells confirms the sample came from the lower respiratory tract. (u) C. The presence of Gram positive cocci has no bearing on the adequacy of the sputum sample. (c) D. The presence of increased polymorphonuclear leukocytes and the absence of squamous epithelial cells are the criteria utilized to evaluate the adequacy of a sputum sample. Ref: (1) 202. Clinical Therapeutics/Psychiatry/Behavioral Medicine A 25 year-old female presents to the emergency department after she collapsed at work. Medical history is significant for a mood disorder that causes her to have "wild mood swings and reckless behavior" according to her husband. She was diagnosed a year ago and since has been treated with several medications. Her symptoms today consist of nausea, vomiting, fatigue, tremor, and hyperreflexia. Lab results show an elevated BUN and creatinine, low sodium and elevated drug levels. All other results are normal. Which of the following medications is most likely the cause of her symptoms? A. Lithium (Lithobid) B. Lorazepam (Ativan) C. Carbamazepine (Tegretol) D. Risperidone (Risperdal)

74

Explanations (c) A. Any sodium loss results in increased lithium levels. Signs and symptoms include vomiting and diarrhea which exacerbate the problem. Tremors, muscle weakness, confusion, vertigo, ataxia, hyperreflexia, rigidity, seizures, and coma may also be present. (u) B. See A for explanation. (u) C. See A for explanation. (u) D. See A for explanation. Ref: (28) 203. Diagnosis/ENT/Ophthalmology A 58 year-old patient presents with spells of dizziness which is described as a spinning sensation. This has occurred several times a day for the last month. The patient also complains of some mild hearing loss, fullness, and a blowing sound in the right ear. Which of the following is the most likely diagnosis? A. Meniere's syndrome B. Labyrinthitis C. Benign paroxysmal positioning vertigo D. Vestibular neuronitis Explanations (c) A. The classic findings of Meniere's syndrome consists of episodic vertigo, with discrete vertigo spells lasting 20 minutes to several hours in association with fluctuating low-frequency sensorineural hearing loss, tinnitus, and a sensation of aural pressure. (u) B. Labyrinthitis is an acute onset of continuous, usually severe vertigo lasting several days to a week, accompanied by hearing loss and tinnitus. (u) C. Benign paroxysmal positioning vertigo is a type of vertigo associated with changes in head position, often rolling over in bed. (u) D. Vestibular neuronitis is a paroxysmal, usually single attack of vertigo that occurs without accompanying impairment of auditory function and will persist for several days to weeks before clearing. Ref: (28) 204. History & Physical/Orthopedics/Rheumatology A 30 year-old diabetic female complains of persistent numbness in her right thumb and forefinger that has been awakening her from sleep for the past week. She is right hand dominant and denies any history of activities involving repetitive motion of the hands. Which of the following is the next step in the evaluation of this patient? A. Electromyogram (EMG) B. Nerve conduction velocity (NCT) C. Phalen maneuver D. Finkelstein test Explanations (u) A. EMG may be performed in a patient who is being considered for surgery. (u) B. Nerve conduction test may be performed if operative intervention is being planned. (c) C. The Phalen maneuver is used to reproduce the symptoms of carpal tunnel syndrome by flexion of the wrist. (u) D. The Finkelstein test is performed when deQuervain's tenosynovitis is suspected. Ref: (26) 205. Clinical Intervention/Hematology A patient with a history of multiple trauma has received a combination of 12 units of packed red blood cells and whole blood while in the emergency department. Later that night it is noted that the patient is beginning to ooze blood from sites of injury and needle punctures. Which of the following is the most appropriate intervention? A. Continue transfusions of packed red blood cells B. Fresh frozen plasma C. Albumin D. Factor VIII concentrate

75

Explanations (h) A. Continued administration of PRBC without replacement of FFP will result in possible volume overload and continued bleeding. (c) B. When a patient receives more than one total blood volume (10 units) without replacement of clotting factors, the patient will continue to bleed without administration of fresh frozen plasma. (u) C. Albumin is used for protein and intravascular volume replacement. (h) D. Factor VIII concentrate is used in the treatment of hemophilia A and would not effect the bleeding in this patient. Ref: (29) 206. Clinical Therapeutics/Gastrointestinal/Nutritional A person presenting with bleeding esophageal varicies should be treated with which of the following while awaiting arrival of endoscopy? A. Carafate (Sucralfate) B. Octreotide (Sandostatin) C. Omeprazole (Prilosec) D. Enoxaparin (Lovenox) Explanations (u) A. Carafate is not indicated in acute gastrointestinal bleeding. (c) B. Octreotide is a vasoacctive drug used in the treatment of GI bleeding as well as somatostatin, vasopressin, and terlipressin. Somatostatin and octreotide are preferred due to safety and less incidence of serious side effects. (u) C. Omeprazole, a proton pump inhibitor, is not indicated in acute gastrointestinal hemorrhage. (h) D. Enoxaparin will increase bleeding and therefore contraindicated in GI bleeding. Ref: (29) 207. Diagnostic Studies/Cardiology Which of the following studies is the best initial diagnostic evaluation to estimate ventricular size and hypertrophy? A. Electrocardiogram (ECG) B. Cardiac CT scan C. Echocardiogram D. Myocardial perfusion imaging Explanations (u) A. ECG is not sensitive or reliable to estimate ventricular size and hypertrophy. (u) B. Cardiac CT scan can detect coronary calcification, but is most sensitive to assess disorders of the aorta. (c) C. Echocardiogram provides the safest and most reliable means to evaluate ventricular size, hypertrophy and function. (u) D. Myocardial perfusion imaging is used for measurement of LV ejection fraction and assess regional wall motion abnormalities. Ref: (11) 208. Diagnosis/Psychiatry/Behavioral Medicine Formications are most commonly associated with which of the following? A. Delusional disorder B. Adverse drug reaction C. Alcohol withdrawal D. Obsessive compulsive disorder Explanations (u) A. The diagnosis of a delusional disorder requires the presence of nonbizzarre delusions of at least one months duration that are not attributed to another disorder. (u) B. Adverse drug reactions are not known to cause formication. (c) C. Formications are the sensation of insects crawling on the skin and is commonly associated with delirium tremens from alcohol withdrawal and cocaine addiction. (u) D. Patients with OCD do not have formication with their disorder. Ref: (14)

76

209. History & Physical/Pulmonology A 24 year-old male presents in respiratory distress and appears quite ill. A Gram stain and culture of the sputum reveals gram-positive cocci in clumps and a chest x-ray reveals multiple patchy infiltrates with some cavitations. Which of the following is most likely to also be found in his medical history? A. IV drug abuse B. Alcohol abuse C. Poor dental hygiene D. HIV positive patient Explanations (c) A. This patient has pneumonia caused by Staphylococcus aureus which is commonly associated with a history of intravenous drug use, influenza epidemics and the hospital setting. (u) B. A history of alcohol abuse is commonly seen with pneumonia caused by Klebsiella pneumoniae. (u) C. Poor dental hygiene is associated with pneumonia caused by anaerobes. (u) D. HIV positive patients are most at risk for development of pneumonia caused by Pneumocystis jiroveci. Ref: (28) 210. Clinical Therapeutics/Neurology Which of the following is first-line treatment for acute inflammatory demyelinating polyneuropathy variant of Guillain-Barre? A. Neostigmine B. IV Solu-Medrol C. Phenytoin D. IV immunoglobulin

Explanations (u) A. Neostigmine may be used for a more chronic variant for symptoms, but not in acute inflammatory variant. (u) B. Treatment with corticosteroids is ineffective and may prolong recovery time. (u) C. Phenytoin may be used for ongoing neuropathic pain, but is not useful in any type of acute neuropathy. (c) D. IV immunoglobulin is effective in patients with Guillain-Barre.

Ref: (28) 211. Diagnostic Studies/Urology/Renal A 21 year-old female presents with dysuria. On examination of the urine, many squamous epithelial cells are noted. Which of the following is the next best step in the evaluation or treatment of this patient? A. Order urine culture and sensitivity B. Obtain renal ultrasound C. Repeat urinalysis with a clean catch sample D. Refer to a nephrologist Explanations (u) A. See C for explanation. (u) B. See C for explanation. (c) C. The presence of many squamous epithelial cells indicates contamination with vaginal flora. The test should be repeated with a clean catch specimen. (u) D. See C for explanation. Ref: (24)

77

212. Diagnosis/Orthopedics/Rheumatology Topic A 42 year-old male sustained a closed left tibial fracture in a fall two days ago. He was treated with a cast for immobilization. Acutely, he developed severe pain in his left leg. Examination reveals the anterolateral aspect of the leg to be exquisitely tender to palpation. The patient has extreme pain with plantar flexion. What is the most likely diagnosis? A. Compartment syndrome B. Deep venous thrombosis C. Osteomyelitis D. Complex regional pain syndrome Explanations (c) A. One of the earliest signs of compartment syndrome is severe pain that occurs with extension of the involved muscles. (u) B. DVT below the knee usually occurs in the posterior compartment and, while potentially painful, should not produce the degree of pain described. (u) C. The pain of osteomyelitis, while potentially severe, should not be greatly exacerbated by manipulation of the overlying muscles. (u) D. Complex regional pain syndrome can present with pain but it is typically a burning pain and often accompanied by vasomotor symptoms. Ref: (27) 213. History & Physical/Gastrointestinal/Nutritional What is considered the most common physical examination finding for intestinal obstruction? A. Distention B. Fluid wave C. Rigidity D. Tenderness Explanations (c) A. Abdominal distention is the most common hallmark of all kinds of intestinal obstructions though its presence can be variable depending on the duration and exact location of the obstruction. (u) B. Fluid wave is noted with ascites, not intestinal obstruction. (u) C. Rigidity is most often minimal to absent except in the most extreme of late presenting strangulated obstructions and is more commonly found early in the course of peritonitis and related phenomenon. (u) D. Tenderness, like rigidity, is not a predominant finding in obstructed bowel until very late in the course of advancing, untreated cases. At this time distention and symptoms of shock will also be present and command the clinical picture. Ref: (7) 214. Clinical Therapeutics/Psychiatry/Behavioral Medicine Which of the following classes of antidepressants is associated with anticholinergic side effects, including cardiac dysrhythmias, dry mouth, sedation, and orthostatic hypotension? A. Selective serotonin reuptake inhibitors B. Monoamine oxidase inhibitors C. Tricyclic antidepressants D. Atypical antidepressants Explanations (u) A. SSRI's do not generally cause anticholinergic side effects. (u) B. MAOI's mainly cause orthostatic hypotension and sympthomimetic effects. (c) C. TCA's have well known anticholinergic effects. (u) D. Atypical antidepressants do not cause anticholinergic side effects. Ref: (28)

78

215. Scientific Concepts/Endocrinology Which of the following hormones primarily inhibits growth hormone secretion from the pituitary gland? A. Testosterone B. Somatostatin C. Dopamine D. Insulin-like growth factor Explanations (u) A. Testosterone has no relationship to growth hormone. (c) B. Somatostatin inhibits the release of growth hormone from the pituitary gland as well as hyperglycemic states. It is therefore useful in the treatment of excessive growth hormone release that occurs with gigantism and acromegaly. (u) C. Dopamine is a vasoactive substance whose main effect is to cause vasoconstriction making it useful in treating hypotension that does not respond to a fluid challenge. (u) D. Insulin-like growth factor parallels the release of growth hormone and growth hormone itself stimulates the release of insulin-like growth factor from the liver and other tissues. Ref: (28) 216. Diagnosis/Cardiology A 44 year-old female presents to clinic for evaluation of a syncopal episode that occurred while walking her dog two days ago. She denies amnesia or head trauma. She has had increasing dyspnea on exertion and pedal edema. Physical examination reveals clubbing of her fingers and central cyanosis. Auscultation of the heart reveals tricuspid insufficiency, widely split second heart sound with a palpable P2. Echocardiogram reveals a large ostium secundum atrial septal defect with bidirectional flow. Which of the following is a secondary complication in this patient? A. Left heart failure B. Ebsteins anomaly C. Tricuspid stenosis D. Pulmonary hypertension Explanations (u) A. This patient would more likely have right heart failure than left heart failure. (u) B. Ebsteins anomaly is apical displacement of the septal tricuspid leaflet and not caused by an ASD. (u) C. Tricuspid stenosis is not caused by an ASD. (c) D. Her symptoms and exam findings are consistent with pulmonary hypertension and in her case, Eisenmengers disease, which is a late finding. Ref: (11) 217. Clinical Therapeutics/Orthopedics/Rheumatology A 27 year-old male presents with gradually worsening low back pain and stiffness for the past two years. His symptoms are worse upon awakening and gradually improve throughout the day. Lumbosacral flexion is less than 50%. Lumbosacral spine films show erosions in the joint line of both sacroiliac joints. HLA-B27 test is positive. Which of the following is the most appropriate first-line medication for this patient? A. Indomethacin (Indocin) B. Methotrexate (Rheumatrex) C. Prednisone (Medrol dose pack) D. Sulfasalazine (Azulfidine) Explanations (c) A. NSAIDs, such as indomethacin, are the mainstay of therapy in ankylosing spondylitis (AS). (u) B. Methotrexate is beneficial in treatment of rheumatoid arthritis, but there is little evidence that it changes the course of disease in ankylosing spondylitis. (u) C. Oral prednisone may be used sparingly in ankylosing spondylitis, but is less useful for this condition because of its association with osteoporosis. (u) D. Sulfasalazine is potentially useful in the treatment of spondyloarthropathies but studies suggest that it is minimally effective in patients with axial disease. Ref: (27)

79

218. Health Maintenance/Pulmonology A mother of a newborn infant presents to the office concerned about reducing the risk of sudden infant death syndrome (SIDS). The infant was delivered at 39 weeks gestation weighing 7 pounds 9 ounces. There is no family history of SIDS and this is her first child. Which of the following is appropriate advice to reduce the risk of SIDS? A. Bottle feeding with soy formula B. Offer a pacifier at nap and bedtimes C. Have the infant sleep in the prone position D. Infant should sleep with the parents to allow close observation Explanations (u) A. Bottle feeding with soy formula does not reduce the risk of SIDS. (c) B. Use of a pacifier during sleeping is a current recommendation to decrease the risk of SIDS. (u) C. Sleeping prone has been consistently shown to increase, not decrease, the risk of SIDS. (u) D. Bed sharing with parents has been shown to increase, not decrease, the risk of SIDS. This risk is increased in infants less than 4 months old and when older children are also present in the bed. Ref: (5) 219. Clinical Intervention/Cardiology Which of the following treatments will most benefit the diabetic patient with two vessel coronary disease? A. Stent placement B. Percutaneous balloon angioplasty C. Medical management D. Coronary artery bypass graft Explanations (u) A. See D for explanation. (u) B. See D for explanation. (u) C. See D for explanation. (c) D. CABG is the treatment of choice in a diabetic with two or three vessel disease. Ref: (11) 220. Clinical Therapeutics/Urology/Renal Which of the following agents can be used as a urinary analgesic? A. Phenazopyridine (Pyridium) B. Oxybutynin (Ditropan) C. Finasteride (Proscar) D. Imipramine (Tofranil) Explanations (c) A. Phenazopyridine is a urinary tract analgesic used in the treatment of urinary tract discomfort. (u) B. Oxybutynin is an antispasmodic and anticholinergic used in the treatment of overactive bladder. (u) C. Finasteride is an alpha-blocker used in the treatment of benign prostatic hypertrophy. (u) D. Imipramine is an anticholinergic used to treat childhood enuresis. Ref: (28) 221. Diagnosis/Pulmonology An infant born at 30 weeks' gestation begins to have respiratory difficulty shortly after birth. Examination reveals rapid, shallow respirations at 80 per minute with associated intercostal retractions, nasal flaring and progressive cyanosis. Chest x-ray reveals the presence of air bronchograms and diffuse bilateral atelectasis. Which of the following is the most likely diagnosis? A. Respiratory distress syndrome B. Spontaneous pneumothorax C. Transient tachypnea syndrome D. Meconium aspiration syndrome

80

Explanations (c) A. Respiratory distress syndrome (hyaline membrane disease) is the most common cause of respiratory distress in a premature infant. This diagnosis is supported by the chest x-ray findings of air bronchograms and diffuse bilateral atelectasis, causing a ground-glass appearance. (u) B. Although spontaneous pneumothorax will present with respiratory distress at birth, the chest x-ray would reveal findings of lung collapse. (u) C. While transient tachypnea syndrome also may present at birth with respiratory distress, the chest x-ray would reveal findings of increased pulmonary vasculature markings, perihilar streaking and fluid in the interlobular fissures. (u) D. Meconium aspiration syndrome usually occurs in term or post-term infants. Typical chest x-ray findings include patchy infiltrates, coarse streaking of both lung fields, increased anteroposterior diameter and flattening of the diaphragm. Ref: (5) 222. Health Maintenance/Orthopedics/Rheumatology A 23 year-old male presents to the emergency department with severe right shoulder pain. He is holding his shoulder in internal rotation. Which of the following would increase your suspicion that he has a posterior glenohumeral dislocation? A. History of a direct posterior blow to the humerus B. History of seizure C. Palpable mass in the anterior axilla D. Palpable space beneath the acromion Explanations (u) A. This is a common mechanism for anterior glenohumeral dislocation. (c) B. Tonic-clonic seizures can create severe contraction of the internal rotator muscles of the shoulder. (u) C. This is a classic finding associated with anterior dislocation. The mass is the humeral head outside of the glenoid. (u) D. This is also a classic finding associated with anterior dislocation and the space is where the humeral head should be in a non-dislocated shoulder. Ref: (26) 223. Clinical Therapeutics/ENT/Ophthalmology A 24 year-old patient presents after a recent vacation. He complains of left ear pain. Physical examination reveals an inflamed external auditory canal and the tympanic membrane can not be visualized. Which of the following is the most appropriate treatment? A. Oral penicillin B. Ciprodex otic drops C. VoSoL drops D. Oral prednisone Explanations (u) A. Oral antibiotics are not indicated in acute otitis externa. (c) B. Topical steroid/antibiotic drops are most useful in acute bacterial otitis externa. (u) C. VoSoL drops are utilized as preventive treatment of otitis externa, but are not effective for treatment. (u) D. Oral prednisone is not indicated in the treatment of acute otitis externa. Ref: (28)

81

224. Scientific Concepts/Gastrointestinal/Nutritional A 33 year-old patient returns from a community picnic that was held on a hot July day. The patient developed severe explosive diarrhea, nausea, vomiting and abdominal cramps six hours after having homemade potato salad and hot dogs. Multiple people who ate at this picnic have similar symptoms. There is no associated fever. What is the most likely causative organism for these symptoms? A. Clostridium perfringens B. Campylobacter jejuni C. Shigella D. Staphylococcus aureus Explanations (u) A. Clostridium perfringens has a 12 to 24 hour incubation period and symptoms are limited to diarrhea and crampy abdominal pain but nausea and vomiting are unusual. (u) B. Campylobacter jejuni, the most common cause of acute bacterial diarrhea, has a prodrome period of 12 to 24-hours with headache, fever, and malaise that is followed by crampy abdominal pain and diarrhea. (u) C. Shigella infection causes bacterial dysentery in tropical regions most commonly and transmission is usually from person to person rather than from food-borne spread. Nausea and vomiting are unusual. (c) D. Staphylococcus aureus is an acute illness caused by the ingestion of enterotoxin-producing organism. It is the only common acute ingestion of a preformed toxin. The clinical picture in the question is the classic presentation for this illness. Foods most responsible for this infection are ham, cream, custard pastries, and mayonnaise-based salads. Ref: (11) 225. Diagnosis/Cardiology A 24 year-old male comes to the clinic with a one week history of pain and swelling that involves the entire right upper extremity. He exercises frequently and has noticed the pain worsening while lifting weights. Examination shows enlarged cutaneous veins over the right anterior chest wall with a palpable cord. His right hand appears dusky. Which of the following is the most likely diagnosis? A. Axillary-subclavian venous thrombosis B. Thromboangiitis obliterans C. Superficial thrombophlebitis of the cephalic vein D. Brachial artery occlusion Explanations (c) A. Axillary-subclavian venous thrombosis can occur in someone who strenuously exercises, has had a central venous catheter or history of venous thrombosis. (u) B. Thromboangiitis obliterans involves the arteries, although the smaller veins can be included and is linked to tobacco use. (u) C. This presentation is not consistent with superficial thrombophlebitis and there is no history of varicosities or previous IVs. (u) D. Symptoms are not consistent with brachial arterial occlusion. Ref: (11)

82

Physician Assistant Education Association

PACKRAT Comment Form

Please fill out this form and return it to: Physician Assistant Education Association 300 N. Washington St., Ste 505 Alexandria, VA 22314 YES NO

1. I found the PACKRAT a useful study tool. Comment:

2. I was able to understand the directions in the Directions and Explanations.

Comment:

3. The feedback provided was easy to understand.

Comment:

4. I know where my weaknesses lie and where to focus my studies.

Comment:

5. The content of the examination was comprehensive.

Comment:

83

6. The examination questions were clearly worded.

Comment:

7. Suggestions for improvement: _________________________________________________________________ _________________________________________________________________ _________________________________________________________________ _________________________________________________________________